Teardowns of electronics in 2019.
You can not select more than 25 topics Topics must start with a letter or number, can include dashes ('-') and can be up to 35 characters long.
 

2047 lines
163 KiB

<!DOCTYPE html>
<html class="html__responsive">
<head>
<title>Why use 2.048V and 4.096 as a reference? - Electrical Engineering Stack Exchange</title>
<link rel="shortcut icon" href="https://cdn.sstatic.net/Sites/electronics/img/favicon.ico?v=575707ea0f4e">
<link rel="apple-touch-icon image_src" href="https://cdn.sstatic.net/Sites/electronics/img/apple-touch-icon.png?v=8ab405a669c4">
<link rel="search" type="application/opensearchdescription+xml" title="Electrical Engineering Stack Exchange" href="/opensearch.xml">
<meta name="viewport" content="width=device-width, height=device-height, initial-scale=1.0, minimum-scale=1.0">
<meta property="og:type" content= "website" />
<meta property="og:url" content="https://electronics.stackexchange.com/questions/364940/why-use-2-048v-and-4-096-as-a-reference"/>
<meta property="og:site_name" content="Electrical Engineering Stack Exchange" />
<meta property="og:image" itemprop="image primaryImageOfPage" content="https://cdn.sstatic.net/Sites/electronics/img/apple-touch-icon@2.png?v=7b89fddaa66b" />
<meta name="twitter:card" content="summary"/>
<meta name="twitter:domain" content="electronics.stackexchange.com"/>
<meta name="twitter:site" content="@StackElectronix" />
<meta name="twitter:creator" content="@StackElectronix" />
<meta name="twitter:title" property="og:title" itemprop="name" content="Why use 2.048V and 4.096 as a reference?" />
<meta name="twitter:description" property="og:description" itemprop="description" content="On many voltage reference ICs (As an example a MAX610x) there seems to be various different reference voltages available (1.25, 1.8, 2.5, 3.3 etc).
What strikes me as odd are the 2.048V and 4.096V " />
<script src="https://ajax.googleapis.com/ajax/libs/jquery/1.12.4/jquery.min.js"></script>
<script src="https://cdn.sstatic.net/Js/stub.en.js?v=f3b7d71d8cf1"></script>
<link rel="stylesheet" type="text/css" href="https://cdn.sstatic.net/Shared/stacks.css?v=1addc9b9addc" >
<link rel="stylesheet" type="text/css" href="https://cdn.sstatic.net/Sites/electronics/primary.css?v=1cd4760fc52a" >
<link rel="alternate" type="application/atom+xml" title="Feed for question &#39;Why use 2.048V and 4.096 as a reference?&#39;" href="/feeds/question/364940">
<meta name="twitter:app:country" content="US" />
<meta name="twitter:app:name:iphone" content="Stack Exchange iOS" />
<meta name="twitter:app:id:iphone" content="871299723" />
<meta name="twitter:app:url:iphone" content="se-zaphod://electronics.stackexchange.com/questions/364940/why-use-2-048v-and-4-096-as-a-reference" />
<meta name="twitter:app:name:ipad" content="Stack Exchange iOS" />
<meta name="twitter:app:id:ipad" content="871299723" />
<meta name="twitter:app:url:ipad" content="se-zaphod://electronics.stackexchange.com/questions/364940/why-use-2-048v-and-4-096-as-a-reference" />
<meta name="twitter:app:name:googleplay" content="Stack Exchange Android">
<meta name="twitter:app:url:googleplay" content="http://electronics.stackexchange.com/questions/364940/why-use-2-048v-and-4-096-as-a-reference">
<meta name="twitter:app:id:googleplay" content="com.stackexchange.marvin">
<script>
StackExchange.ready(function () {
StackExchange.using("postValidation", function () {
StackExchange.postValidation.initOnBlurAndSubmit($('#post-form'), 2, 'answer');
});
StackExchange.question.init({showAnswerHelp:true,totalCommentCount:2,shownCommentCount:2,highlightColor:'#F4A83D',backgroundColor:'#FFF',questionId:364940});
styleCode();
StackExchange.realtime.subscribeToQuestion('135', '364940');
StackExchange.using("gps", function () { StackExchange.gps.trackOutboundClicks('#content', '.post-text'); });
});
</script>
<script type="text/x-mathjax-config">
MathJax.Hub.Config({"HTML-CSS": { preferredFont: "TeX", availableFonts: ["STIX","TeX"], linebreaks: { automatic:true }, EqnChunk: (MathJax.Hub.Browser.isMobile ? 10 : 50) },
tex2jax: { inlineMath: [ ["\\$", "\\$"] ], displayMath: [ ["$$","$$"], ["\\[", "\\]"] ], processEscapes: true, ignoreClass: "tex2jax_ignore|dno" },
TeX: {
extensions: ["begingroup.js"],
noUndefined: { attributes: { mathcolor: "red", mathbackground: "#FFEEEE", mathsize: "90%" } },
Macros: { href: "{}" }
},
messageStyle: "none",
styles: { ".MathJax_Display, .MathJax_Preview, .MathJax_Preview > *": { "background": "inherit" } },
SEEditor: "mathjaxEditing"
});
</script>
<script src="https://cdnjs.cloudflare.com/ajax/libs/mathjax/2.7.5/MathJax.js?config=TeX-AMS_HTML-full"></script>
<script>
StackExchange.ready(function () {
StackExchange.realtime.init('wss://qa.sockets.stackexchange.com');
StackExchange.realtime.subscribeToReputationNotifications('135');
StackExchange.realtime.subscribeToTopBarNotifications('135');
});
</script>
<script>
StackExchange.init({"locale":"en","serverTime":1574915457,"routeName":"Questions/Show","stackAuthUrl":"https://stackauth.com","networkMetaHostname":"meta.stackexchange.com","site":{"name":"Electrical Engineering Stack Exchange","description":"Q&A for electronics and electrical engineering professionals, students, and enthusiasts","isNoticesTabEnabled":true,"enableNewTagCreationWarning":false,"insertSpaceAfterNameTabCompletion":false,"id":135,"childUrl":"https://electronics.meta.stackexchange.com","negativeVoteScoreFloor":null,"enableSocialMediaInSharePopup":true,"protocol":"https"},"user":{"fkey":"6067184cd28298c9cd1163745c0315d13a2a3e1c0e8abc4ae1db30773cc2fc93","tid":"e2743a07-4470-d92e-448a-1ec46514bb72","rep":0,"isAnonymous":true,"isAnonymousNetworkWide":true},"realtime":{"newest":true,"active":true,"tagged":true,"staleDisconnectIntervalInHours":0},"events":{"postType":{"question":1},"postEditionSection":{"title":1,"body":2,"tags":3}},"story":{"minCompleteBodyLength":75,"likedTagsMaxLength":300,"dislikedTagsMaxLength":300},"jobPreferences":{"maxNumDeveloperRoles":2,"maxNumIndustries":4},"svgIconPath":"https://cdn.sstatic.net/Img/svg-icons","svgIconHash":"53fe1f91bb82"}, {"userProfile":{},"userMessaging":{"showNewFeatureNotice":true},"tags":{},"snippets":{"renderDomain":"stacksnippets.net"},"slack":{"sidebarAdDismissCookie":"slack-sidebar-ad"},"site":{"allowImageUploads":true,"enableImgurHttps":true,"enableUserHovercards":true,"forceHttpsImages":true,"styleCode":true},"questions":{},"paths":{},"monitoring":{"clientTimingsAbsoluteTimeout":30000,"clientTimingsDebounceTimeout":1000},"mentions":{"maxNumUsersInDropdown":50},"markdown":{"asteriskIntraWordEmphasis":true},"flags":{"allowRetractingCommentFlags":true,"allowRetractingFlags":true},"comments":{},"accounts":{"currentPasswordRequiredForChangingStackIdPassword":true}});
StackExchange.using.setCacheBreakers({"js/prettify-full.en.js":"81a7c250c473","js/moderator.en.js":"188e06c3da4f","js/full-anon.en.js":"969c2baf217b","js/full.en.js":"99d6d7ce6a4c","js/wmd.en.js":"2cc6c5f01ed6","js/mobile.en.js":"b2b68dcf671f","js/help.en.js":"bcaf279b6d04","js/tageditor.en.js":"0acb93384417","js/tageditornew.en.js":"7fb9cacaa5b0","js/inline-tag-editing.en.js":"06aa7aefcf3f","js/revisions.en.js":"c096ee06038c","js/review.en.js":"d9f82f0bc8e7","js/tagsuggestions.en.js":"a9ff449efcdc","js/post-validation.en.js":"d9783566f904","js/explore-qlist.en.js":"be3b9d8380ea","js/events.en.js":"c07c1cfae126","js/keyboard-shortcuts.en.js":"62c549661637","js/adops.en.js":"6b9883f0531e","js/begin-edit-event.en.js":"8f05bb4ed432","js/ask.en.js":"43f1476c5dac","js/question-editor.en.js":"","js/mathjax-editing.en.js":"199e0b9d3fd5"});
StackExchange.using("gps", function() {
StackExchange.gps.init(true);
});
</script>
<noscript id="noscript-css"><style>body,.top-bar{margin-top:1.9em}</style></noscript>
</head>
<body class="question-page unified-theme">
<div id="notify-container"></div>
<div id="custom-header"></div>
<header class="top-bar js-top-bar top-bar__network _fixed">
<div class="wmx12 mx-auto grid ai-center h100" role="menubar">
<div class="-main grid--cell">
<a href="#" class="left-sidebar-toggle p0 ai-center jc-center js-left-sidebar-toggle" role="menuitem" aria-haspopup="true" aria-controls="left-sidebar" aria-expanded="false"><span class="ps-relative"></span></a>
<div class="topbar-dialog leftnav-dialog js-leftnav-dialog dno">
<div class="left-sidebar js-unpinned-left-sidebar" data-can-be="left-sidebar" data-is-here-when="sm"></div>
</div>
<a href="#" class="-logo js-gps-track js-network-logo network-logo"
data-gps-track="stack_exchange_popup.show" role="menuitem" aria-haspopup="true" aria-controls="topbar-network-logo-dialog" aria-expanded="false">
<svg aria-hidden="true" class="svg-icon native mtn1 iconLogoSEAlternativeSm" width="107" height="15" viewBox="0 0 107 15"><path d="M48.41 11.93l-1.96-3.2-1.04 1.16v2.04h-1.42V2.18h1.42v6.01L48.14 5h1.72l-2.44 2.7 2.74 4.22h-1.75zm-7.06.08c-1.59 0-3.14-.96-3.14-3.56s1.55-3.54 3.14-3.54c.97 0 1.65.27 2.31.97l-.97.93c-.44-.48-.79-.66-1.34-.66s-1 .22-1.3.62c-.31.38-.42.87-.42 1.68 0 .81.1 1.32.41 1.7.3.4.76.62 1.3.62.56 0 .9-.18 1.35-.66l.97.92c-.66.7-1.34.98-2.31.98zm-5.66-3.15h-1.65c-.83 0-1.26.37-1.26 1s.4.99 1.3.99c.53 0 .93-.04 1.3-.4.22-.2.31-.53.31-1.03v-.56zm.03 3.07v-.63c-.51.5-1 .71-1.87.71-.87 0-1.46-.2-1.89-.63a2.1 2.1 0 0 1-.55-1.49c0-1.16.82-2 2.42-2h1.86v-.5c0-.87-.44-1.3-1.54-1.3-.77 0-1.15.18-1.54.68l-.92-.86c.66-.77 1.35-1 2.52-1 1.93 0 2.9.8 2.9 2.38v4.64h-1.39zm-5.9 0c-1.32 0-1.93-.93-1.93-1.93V6.18h-.8V5.1h.8V3h1.41v2.1h1.36v1.07H29.3v3.75c0 .5.25.81.78.81h.58v1.2h-.85zm-6.33.08c-1.48 0-2.55-.34-3.49-1.28l1-.98c.72.72 1.51.94 2.52.94 1.3 0 2.04-.55 2.04-1.5 0-.42-.13-.78-.39-1.01-.25-.23-.5-.33-1.08-.41l-1.16-.17a3.4 3.4 0 0 1-1.88-.78 2.41 2.41 0 0 1-.72-1.86c0-1.7 1.25-2.86 3.3-2.86 1.3 0 2.22.33 3.07 1.1l-.96.94a2.92 2.92 0 0 0-2.15-.75c-1.16 0-1.8.65-1.8 1.52 0 .35.1.67.37.9.25.22.65.38 1.11.45l1.13.17c.91.13 1.42.35 1.84.72.54.47.8 1.17.8 2 0 1.8-1.48 2.86-3.55 2.86z" fill="#fff"/><path d="M104.16 7.09c-.2-.42-.6-.74-1.2-.74s-.99.32-1.18.74c-.1.25-.15.44-.16.75h2.7a2 2 0 0 0-.16-.75zm-2.54 1.96c0 .9.56 1.57 1.55 1.57.78 0 1.16-.21 1.61-.66l1.08 1.04a3.4 3.4 0 0 1-2.7 1.11c-1.68 0-3.29-.76-3.29-3.62 0-2.3 1.26-3.6 3.1-3.6 1.97 0 3.1 1.44 3.1 3.37v.79h-4.45zm-5.48-2.57C95.1 6.48 95 7.37 95 8.3c0 .94.1 1.85 1.15 1.85 1.05 0 1.18-.91 1.18-1.85 0-.93-.13-1.82-1.18-1.82zm-.17 8.22c-1.1 0-1.84-.21-2.58-.92l1.1-1.11c.4.38.8.54 1.4.54 1.06 0 1.43-.74 1.43-1.46v-.72c-.47.51-1 .7-1.7.7-.69 0-1.29-.23-1.68-.62-.67-.66-.73-1.57-.73-2.8 0-1.24.06-2.13.73-2.8.4-.39 1-.62 1.7-.62.75 0 1.24.2 1.73.75v-.67h1.72v6.8c0 1.7-1.21 2.93-3.12 2.93zm-5.76-2.67V7.76c0-.96-.61-1.28-1.17-1.28-.56 0-1.18.32-1.18 1.28v4.27h-1.78V4.97h1.73v.65a2.44 2.44 0 0 1 1.78-.73c.7 0 1.28.23 1.67.62.58.57.73 1.24.73 2v4.52H90.2zm-7.1-2.98h-1.4c-.64 0-1 .3-1 .8 0 .49.33.81 1.02.81.5 0 .8-.04 1.12-.34.2-.17.26-.46.26-.89v-.38zm.04 2.98v-.6c-.48.47-.93.67-1.74.67-.8 0-1.4-.2-1.82-.62-.38-.4-.58-.97-.58-1.59 0-1.12.77-2.05 2.42-2.05h1.68V7.5c0-.77-.38-1.11-1.32-1.11-.68 0-1 .16-1.37.58l-1.13-1.1c.7-.75 1.38-.97 2.57-.97 1.99 0 3.02.84 3.02 2.5v4.64h-1.73zm-6.93 0v-4.3c0-.94-.6-1.25-1.15-1.25-.56 0-1.15.32-1.15 1.24v4.31h-1.77V2.38h1.77v3.24a2.35 2.35 0 0 1 1.7-.73c1.56 0 2.38 1.08 2.38 2.57v4.57h-1.78zm-6.96.08c-1.42 0-3.18-.76-3.18-3.62 0-2.85 1.76-3.6 3.18-3.6.98 0 1.72.3 2.34.95l-1.2 1.2c-.36-.4-.68-.56-1.14-.56-.42 0-.75.14-1.01.46-.27.33-.4.8-.4 1.55s.13 1.24.4 1.58c.26.3.59.46 1 .46.47 0 .79-.16 1.15-.56l1.2 1.18c-.62.65-1.36.96-2.34.96zm-5.53-.08l-1.3-2.11-1.3 2.11H59l2.45-3.6-2.35-3.46h2.12L62.42 7l1.21-2.02h2.13L63.4 8.43l2.46 3.6h-2.13zm-11.75 0V2.06h6.6V3.8h-4.65v2.33h3.96v1.74h-3.96v2.42h4.65v1.74h-6.6z" fill="#2F96E8"/><path d="M0 3c0-1.1.9-2 2-2h8a2 2 0 0 1 2 2H0z" fill="#8FD8F7"/><path d="M12 10H0c0 1.1.9 2 2 2h5v3l3-3a2 2 0 0 0 2-2z" fill="#155397"/><path fill="#46A2D9" d="M0 4h12v2H0z"/><path fill="#2D6DB5" d="M0 7h12v2H0z"/></svg>
</a>
<div class="topbar-dialog network-logo-dialog js-network-logo-dialog dno" id="topbar-network-logo-dialog" role="dialog" aria-labelledby="topbar-network-logo-dialog-title" aria-describedby="topbar-network-logo-dialog-body">
<div class="dialog-content">
<h4 class="bold" id="topbar-network-logo-dialog-title">Stack Exchange Network</h4>
<p id="topbar-network-logo-dialog-body">
Stack Exchange network consists of 175 Q&amp;A communities including <a href="https://stackoverflow.com">Stack Overflow</a>, the largest, most trusted online community for developers to learn, share their knowledge, and build their careers.
</p>
<a class="btn-secondary" href="https://stackexchange.com"
data-gps-track="stack_exchange_popup.click">Visit Stack Exchange</a>
<button class="icon-close js-close-button s-btn s-btn__unset" aria-label="Close"><svg aria-hidden="true" class="svg-icon iconClear" width="18" height="18" viewBox="0 0 18 18"><path d="M15 4.41L13.59 3 9 7.59 4.41 3 3 4.41 7.59 9 3 13.59 4.41 15 9 10.41 13.59 15 15 13.59 10.41 9 15 4.41z"/></svg></button>
</div>
</div>
</div>
<form id="search" role="search" action=/search method="get" class="grid--cell fl-grow1 searchbar px12 js-searchbar " autocomplete="off">
<div class="ps-relative">
<input name="q"
type="text"
placeholder="Search on Electrical Engineering…"
value=""
autocomplete="off"
maxlength="240"
class="s-input s-input__search js-search-field "
aria-label="Search"
aria-controls="top-search"
data-controller="s-popover"
data-action="focus->s-popover#show"
data-s-popover-placement="bottom-start"/>
<svg aria-hidden="true" class="svg-icon s-input-icon s-input-icon__search iconSearch" width="18" height="18" viewBox="0 0 18 18"><path d="M18 16.5l-5.14-5.18h-.35a7 7 0 1 0-1.19 1.19v.35L16.5 18l1.5-1.5zM12 7A5 5 0 1 1 2 7a5 5 0 0 1 10 0z"/></svg>
<div class="s-popover p0 wmx100 wmn4 sm:wmn-initial js-top-search-popover s-popover--arrow__tl" id="top-search" role="menu">
<div class="js-spinner p24 grid ai-center jc-center d-none">
<div class="s-spinner s-spinner__sm fc-orange-400">
<div class="v-visible-sr">Loading…</div>
</div>
</div>
<span class="v-visible-sr js-screen-reader-info"></span>
<div class="js-ac-results overflow-y-auto hmx3 d-none"></div>
<div class="js-search-hints" aria-describedby="Tips for searching"></div>
</div>
</div>
</form>
<ol class="overflow-x-auto ml-auto -secondary grid ai-center list-reset h100 user-logged-out" role="presentation">
<li class="-item searchbar-trigger"><a href="#" class="-link js-searchbar-trigger" role="button" aria-label="Search" aria-haspopup="true" aria-controls="search" title="Click to show search"><svg aria-hidden="true" class="svg-icon iconSearch" width="18" height="18" viewBox="0 0 18 18"><path d="M18 16.5l-5.14-5.18h-.35a7 7 0 1 0-1.19 1.19v.35L16.5 18l1.5-1.5zM12 7A5 5 0 1 1 2 7a5 5 0 0 1 10 0z"/></svg></a></li>
<li class="-item inbox-button-item">
<a href="#" class="-link js-inbox-button"
aria-label="Inbox" title="Recent inbox messages" role="menuitem" aria-haspopup="true" aria-expanded="false" data-unread-count="0">
<svg aria-hidden="true" class="svg-icon iconInbox" width="20" height="18" viewBox="0 0 20 18"><path d="M4.63 1h10.56a2 2 0 0 1 1.94 1.35L20 10.79V15a2 2 0 0 1-2 2H2a2 2 0 0 1-2-2v-4.21l2.78-8.44c.25-.8 1-1.36 1.85-1.35zm8.28 12l2-2h2.95l-2.44-7.32a1 1 0 0 0-.95-.68H5.35a1 1 0 0 0-.95.68L1.96 11h2.95l2 2h6z"/></svg>
<span class="indicator-badge js-unread-count _important d-none">0</span>
</a>
</li>
<li class="-item achievements-button-item">
<a href="#" class="-link js-achievements-button" data-unread-class="_highlighted-positive"
aria-label="Achievements" title="Recent achievements: reputation, badges, and privileges earned" role="menuitem" aria-haspopup="true" aria-expanded="false" data-unread-count="0" data-lit-up="false">
<svg aria-hidden="true" class="svg-icon iconAchievements" width="18" height="18" viewBox="0 0 18 18"><path d="M15 2V1H3v1H0v4c0 1.6 1.4 3 3 3v1c.4 1.5 3 2.6 5 3v2H5s-1 1.5-1 2h10c0-.4-1-2-1-2h-3v-2c2-.4 4.6-1.5 5-3V9c1.6-.2 3-1.4 3-3V2h-3zM3 7c-.5 0-1-.5-1-1V4h1v3zm8.4 2.5L9 8 6.6 9.4l1-2.7L5 5h3l1-2.7L10 5h2.8l-2.3 1.8 1 2.7h-.1zM16 6c0 .5-.5 1-1 1V4h1v2z"/></svg>
<span class="indicator-badge js-unread-count _positive d-none">+0</span>
</a>
</li>
<li class="-item help-button-item">
<a href="#" class="-link js-help-button" title="Help Center and other resources" role="menuitem" aria-haspopup="true" aria-controls="topbar-help-dialog"
data-ga="[&quot;top navigation&quot;,&quot;help menu click&quot;,null,null,null]"><svg aria-hidden="true" class="svg-icon iconHelp" width="18" height="18" viewBox="0 0 18 18"><path d="M9 1a8 8 0 1 0 0 16A8 8 0 0 0 9 1zm.81 12.13c-.02.71-.55 1.15-1.24 1.13-.66-.02-1.17-.49-1.15-1.2.02-.72.56-1.18 1.22-1.16.7.03 1.2.51 1.17 1.23zM11.77 8c-.3.34-.65.65-1.02.91l-.53.37c-.26.2-.42.43-.5.69a4 4 0 0 0-.09.75c0 .05-.03.16-.18.16H7.88c-.16 0-.18-.1-.18-.15.03-.66.12-1.21.4-1.66.4-.49.88-.9 1.43-1.22.16-.12.28-.25.38-.39a1.34 1.34 0 0 0 .02-1.71c-.24-.31-.51-.46-1.03-.46-.51 0-.8.26-1.02.6-.21.33-.18.73-.18 1.1H5.75c0-1.38.35-2.25 1.1-2.76.52-.35 1.17-.5 1.93-.5 1 0 1.79.18 2.49.71.64.5.98 1.18.98 2.12 0 .57-.2 1.05-.48 1.44z"/></svg></a>
</li>
<div class="topbar-dialog help-dialog js-help-dialog dno" id="topbar-help-dialog" role="menu">
<div class="modal-content">
<ul>
<li>
<a href="/tour" class="js-gps-track" data-gps-track="help_popup.click({ item_type:1 })"
data-ga="[&quot;top navigation&quot;,&quot;tour submenu click&quot;,null,null,null]">
Tour
<span class="item-summary">
Start here for a quick overview of the site
</span>
</a>
</li>
<li>
<a href="/help" class="js-gps-track"
data-gps-track="help_popup.click({ item_type:4 })"
data-ga="[&quot;top navigation&quot;,&quot;help center&quot;,null,null,null]">
Help Center
<span class="item-summary">
Detailed answers to any questions you might have
</span>
</a>
</li>
<li>
<a href="https://electronics.meta.stackexchange.com" class="js-gps-track" data-gps-track="help_popup.click({ item_type:2 })"
data-ga="[&quot;top navigation&quot;,&quot;meta submenu click&quot;,null,null,null]">
Meta
<span class="item-summary">
Discuss the workings and policies of this site
</span>
</a>
</li>
<li>
<a href="https://stackoverflow.com/company/about" class="js-gps-track" data-gps-track="help_popup.click({ item_type:6 })"
data-ga="[&quot;top navigation&quot;,&quot;about us submenu click&quot;,null,null,null]">
About Us
<span class="item-summary">
Learn more about Stack Overflow the company
</span>
</a>
</li>
<li>
<a href="https://www.stackoverflowbusiness.com/?ref=topbar_help" class="js-gps-track" data-gps-track="help_popup.click({ item_type:7 })"
data-ga="[&quot;top navigation&quot;,&quot;business submenu click&quot;,null,null,null]">
Business
<span class="item-summary">
Learn more about hiring developers or posting ads with us
</span>
</a>
</li>
</ul>
</div>
</div>
<li class="-item site-switcher-item">
<a href="https://stackexchange.com" class="-link js-site-switcher-button js-gps-track" data-gps-track="site_switcher.show"
aria-label="Site switcher"
title="A list of all 175 Stack Exchange sites"
role="menuitem" aria-haspopup="true" aria-expanded="false"
data-ga="[&quot;top navigation&quot;,&quot;stack exchange click&quot;,null,null,null]">
<svg aria-hidden="true" class="svg-icon iconStackExchange" width="18" height="18" viewBox="0 0 18 18"><path d="M15 1H3a2 2 0 0 0-2 2v2h16V3a2 2 0 0 0-2-2zM1 13c0 1.1.9 2 2 2h8v3l3-3h1a2 2 0 0 0 2-2v-2H1v2zm16-7H1v4h16V6z"/></svg>
</a>
</li>
<li class="-ctas">
<a href="https://electronics.stackexchange.com/users/login?ssrc=head&returnurl=https%3a%2f%2felectronics.stackexchange.com%2fquestions%2f364940%2fwhy-use-2-048v-and-4-096-as-a-reference" class="login-link s-btn btn-topbar-clear py8 js-gps-track" rel="nofollow"
data-gps-track="login.click" data-ga="[&quot;top navigation&quot;,&quot;login button click&quot;,null,null,null]">Log in</a>
<a href="https://electronics.stackexchange.com/users/signup?ssrc=head&returnurl=https%3a%2f%2felectronics.stackexchange.com%2fquestions%2f364940%2fwhy-use-2-048v-and-4-096-as-a-reference" class="login-link s-btn s-btn__primary py8 btn-topbar-primary" rel="nofollow" data-ga="[&quot;sign up&quot;,&quot;Sign Up Navigation&quot;,&quot;Header&quot;,null,null]">Sign up</a>
</li>
<li class="js-topbar-dialog-corral" role="presentation">
<div class="topbar-dialog siteSwitcher-dialog dno" role="menu">
<div class="header">
<h3><a href="https://electronics.stackexchange.com">current community</a>
</h3>
</div>
<div class="modal-content current-site-container">
<ul class="current-site">
<li class="grid">
<div class="fl1">
<a href="https://electronics.stackexchange.com"
class="current-site-link site-link js-gps-track grid gs8 gsx"
data-id="135"
data-gps-track="site_switcher.click({ item_type:3 })">
<div class="favicon favicon-electronics site-icon grid--cell" title="Electrical Engineering"></div>
<span class="grid--cell fl1">
Electrical Engineering
</span>
</a>
</div>
<div class="related-links">
<a href="https://electronics.stackexchange.com/help" class="js-gps-track" data-gps-track="site_switcher.click({ item_type:14 })"
>help</a>
<a href="https://chat.stackexchange.com?tab=site&amp;host=electronics.stackexchange.com" class="js-gps-track" data-gps-track="site_switcher.click({ item_type:6 })"
>chat</a>
</div>
</li>
<li class="related-site grid">
<div class="L-shaped-icon-container">
<span class="L-shaped-icon"></span>
</div>
<a href="https://electronics.meta.stackexchange.com"
class="site-link js-gps-track grid gs8 gsx"
data-id="137"
data-gps-track="site.switch({ target_site:137, item_type:3 }),site_switcher.click({ item_type:4 })">
<div class="favicon favicon-electronicsmeta site-icon grid--cell" title="Electrical Engineering Meta"></div>
<span class="grid--cell fl1">
Electrical Engineering Meta
</span>
</a>
</li>
</ul>
</div>
<div class="header" id="your-communities-header">
<h3>
your communities </h3>
</div>
<div class="modal-content" id="your-communities-section">
<div class="call-to-login">
<a href="https://electronics.stackexchange.com/users/signup?ssrc=site_switcher&amp;returnurl=https%3a%2f%2felectronics.stackexchange.com%2fquestions%2f364940%2fwhy-use-2-048v-and-4-096-as-a-reference" class="login-link js-gps-track" data-gps-track="site_switcher.click({ item_type:10 })"
>Sign up</a> or <a href="https://electronics.stackexchange.com/users/login?ssrc=site_switcher&amp;returnurl=https%3a%2f%2felectronics.stackexchange.com%2fquestions%2f364940%2fwhy-use-2-048v-and-4-096-as-a-reference" class="login-link js-gps-track" data-gps-track="site_switcher.click({ item_type:11 })"
>log in</a> to customize your list.
</div>
</div>
<div class="header">
<h3><a href="https://stackexchange.com/sites">more stack exchange communities</a>
</h3>
<a href="https://stackoverflow.blog" class="fr">company blog</a>
</div>
<div class="modal-content">
<div class="child-content"></div>
</div>
</div>
</li>
</ol>
</div>
</header>
<div id="js-gdpr-consent-banner" class="p16 bg-black-700 ff-sans fc-white ps-fixed b0 l0 r0 z-banner" role="banner" aria-hidden="false">
<div class="wmx8 mx-auto grid jc-center" role="alertdialog" aria-describedby="notice-message">
<div class="grid--cell mr12" aria-label="notice-message">
<p class="mb0 lh-lg">
By using our site, you acknowledge that you have read and understand our <a class="s-link s-link__inherit td-underline fc-white" target="_blank" href="https://stackoverflow.com/legal/cookie-policy">Cookie Policy</a>, <a class="s-link s-link__inherit td-underline fc-white" target="_blank" href="https://stackoverflow.com/legal/privacy-policy">Privacy Policy</a>, and our <a class="s-link s-link__inherit td-underline fc-white" target="_blank" href="https://stackoverflow.com/legal/terms-of-service/public">Terms of Service</a>.
</p>
</div>
<a class="grid--cell fc-white js-notice-close" aria-label="notice-dismiss">
<svg aria-hidden="true" class="svg-icon m0 iconClear" width="18" height="18" viewBox="0 0 18 18"><path d="M15 4.41L13.59 3 9 7.59 4.41 3 3 4.41 7.59 9 3 13.59 4.41 15 9 10.41 13.59 15 15 13.59 10.41 9 15 4.41z"/></svg>
</a>
</div>
</div>
<script>
StackExchange.ready(function () { StackExchange.topbar.init(); });
StackExchange.scrollPadding.setPaddingTop(50, 10); </script>
<header class="site-header">
<div class="site-header--container">
<a class="site-header--link d-flex ai-center fs-headline1 fw-bold" href="https://electronics.stackexchange.com">
<img class="h-auto wmx100" src="https://cdn.sstatic.net/Sites/electronics/img/logo.svg?v=edbe2da2a113" alt="Electrical Engineering">
</a>
</div>
</header>
<div class="container">
<div id="left-sidebar" data-is-here-when="md lg" class="left-sidebar js-pinned-left-sidebar">
<div class="left-sidebar--sticky-container js-sticky-leftnav">
<nav role="navigation">
<ol class="nav-links">
<li>
<a
href="/"
class="pl8 js-gps-track nav-links--link"
data-gps-track="top_nav.click({is_current:false, location:2, destination:8})">
Home </a>
</li>
<li>
<ol class="nav-links">
<li class=" youarehere">
<a id="nav-questions"
href="/questions"
class="pl8 js-gps-track nav-links--link"
data-gps-track="top_nav.click({is_current:true, location:2, destination:1})">
Questions </a>
</li>
<li>
<a id="nav-tags"
href="/tags"
class="pl8 js-gps-track nav-links--link"
data-gps-track="top_nav.click({is_current:false, location:2, destination:2})">
Tags </a>
</li>
<li>
<a id="nav-users"
href="/users"
class="pl8 js-gps-track nav-links--link"
data-gps-track="top_nav.click({is_current:false, location:2, destination:3})">
Users </a>
</li>
<li>
<a id="nav-unanswered"
href="/unanswered"
class="pl8 js-gps-track nav-links--link"
data-gps-track="top_nav.click({is_current:false, location:2, destination:5})">
Unanswered </a>
</li>
</ol>
</li>
</ol>
</nav>
</div>
</div>
<div id="content" class="">
<div itemprop="mainEntity" itemscope itemtype="http://schema.org/Question">
<link itemprop="image" href="https://cdn.sstatic.net/Sites/electronics/img/apple-touch-icon.png?v=8ab405a669c4">
<div class="inner-content clearfix">
<div id="question-header" class="grid sm:fd-column">
<h1 itemprop="name" class="grid--cell fs-headline1 fl1 ow-break-word mb8"><a href="/questions/364940/why-use-2-048v-and-4-096-as-a-reference" class="question-hyperlink">Why use 2.048V and 4.096 as a reference?</a></h1>
<div class="ml12 aside-cta grid--cell print:d-none sm:ml0 sm:mb12 sm:order-first sm:as-end">
<a href="/questions/ask" class="ws-nowrap s-btn s-btn__primary" >
Ask Question
</a>
</div>
</div>
<div class="grid fw-wrap pb8 mb16 bb bc-black-2">
<div class="grid--cell ws-nowrap mr16 mb8" title="2018-03-27 14:53:56Z">
<span class="fc-light mr2">Asked</span>
<time itemprop="dateCreated" datetime="2018-03-27T14:53:56">1 year, 8 months ago</time>
</div>
<div class="grid--cell ws-nowrap mr16 mb8">
<span class="fc-light mr2">Active</span>
<a href="?lastactivity" class="s-link s-link__inherit" title="2018-03-29 11:22:28Z">1 year, 8 months ago</a>
</div>
<div class="grid--cell ws-nowrap mb8" title="Viewed 6,222 times">
<span class="fc-light mr2">Viewed</span>
6k times
</div>
</div>
<div id="mainbar" role="main" aria-label="question and answers">
<div class="question" data-questionid="364940" id="question">
<style>.everyoneloves__top-leaderboard:empty,.everyoneloves__mid-leaderboard:empty,.everyoneloves__bot-mid-leaderboard:empty{
margin-bottom:0;
}
</style>
<div id="dfp-tlb" class="everyonelovesstackoverflow everyoneloves__top-leaderboard"></div>
<div class="post-layout">
<div class="votecell post-layout--left">
<div class="js-voting-container grid fd-column ai-stretch gs4 fc-black-200" data-post-id="364940">
<button class="js-vote-up-btn grid--cell s-btn s-btn__unset c-pointer" title="This question shows research effort; it is useful and clear" aria-pressed="false" aria-label="up vote" data-selected-classes="fc-theme-primary"><svg aria-hidden="true" class="svg-icon m0 iconArrowUpLg" width="36" height="36" viewBox="0 0 36 36"><path d="M2 26h32L18 10 2 26z"/></svg></button>
<div class="js-vote-count grid--cell fc-black-500 fs-title grid fd-column ai-center" itemprop="upvoteCount" data-value="25">25</div>
<button class="js-vote-down-btn grid--cell s-btn s-btn__unset c-pointer" title="This question does not show any research effort; it is unclear or not useful" aria-pressed="false" aria-label="down vote" data-selected-classes="fc-theme-primary"><svg aria-hidden="true" class="svg-icon m0 iconArrowDownLg" width="36" height="36" viewBox="0 0 36 36"><path d="M2 10h32L18 26 2 10z"/></svg></button>
<button class="js-favorite-btn s-btn s-btn__unset c-pointer py4 js-gps-track" aria-pressed="false" aria-label="favorite (2)" data-selected-classes="fc-yellow-600"
data-gps-track="post.click({ item: 1, priv: 0, post_type: 1 })">
<svg aria-hidden="true" class="svg-icon iconStar" width="18" height="18" viewBox="0 0 18 18"><path d="M9 12.65l-5.29 3.63 1.82-6.15L.44 6.22l6.42-.17L9 0l2.14 6.05 6.42.17-5.1 3.9 1.83 6.16L9 12.65z"/></svg>
<div class="js-favorite-count mt4" data-value="2">2</div>
</button>
</div>
</div>
<div class="postcell post-layout--right">
<span class="d-none">\$\begingroup\$</span>
<div class="post-text" itemprop="text">
<p>On many voltage reference ICs (As an example a <a href="https://datasheets.maximintegrated.com/en/ds/MAX6100-MAX6107.pdf" rel="noreferrer">MAX610x</a>) there seems to be various different reference voltages available (1.25, 1.8, 2.5, 3.3 etc). </p>
<p>What strikes me as odd are the 2.048V and 4.096V references. Why do we use references at those voltages instead of just simply 2V and 4V which would surely be easier to use mathematically? </p>
</div>
<div class="post-taglist grid gs4 gsy fd-column">
<div class="grid ps-relative d-block">
<a href="/questions/tagged/voltage-reference" class="post-tag" title="show questions tagged &#39;voltage-reference&#39;" rel="tag">voltage-reference</a>
</div>
</div>
<div class="mb0 ">
<div class="mt16 grid gs8 gsy fw-wrap jc-end ai-start pt4">
<div class="grid--cell mr16" style="flex: 1 1 100px;">
<div class="post-menu"><a href="/q/364940" itemprop="url"
class="js-share-link js-gps-track"
title="short permalink to this question"
data-gps-track="post.click({ item: 2, priv: 0, post_type: 1 })"
data-controller="se-share-sheet"
data-se-share-sheet-title="Share a link to this question"
data-se-share-sheet-subtitle=""
data-se-share-sheet-post-type="question"
data-se-share-sheet-social="facebook twitter "
data-se-share-sheet-location="1"
data-s-popover-placement="bottom-start">share</a><span class="lsep">|</span><a href="/posts/364940/edit" class="suggest-edit-post js-gps-track" data-gps-track="post.click({ item: 6, priv: 0, post_type: 1 })" title="">improve this question</a></div>
</div>
<div class="post-signature owner grid--cell">
<div class="user-info user-hover">
<div class="user-action-time">
asked <span title="2018-03-27 14:53:56Z" class="relativetime">Mar 27 '18 at 14:53</span>
</div>
<div class="user-gravatar32">
<a href="/users/138493/mcg"><div class="gravatar-wrapper-32"><img src="https://i.stack.imgur.com/Hot2G.jpg?s=32&amp;g=1" alt="" width="32" height="32" class="bar-sm"></div></a>
</div>
<div class="user-details" itemprop="author" itemscope itemtype="http://schema.org/Person">
<a href="/users/138493/mcg">MCG</a><span class="d-none" itemprop="name">MCG</span>
<div class="-flair">
<span class="reputation-score" title="reputation score 10,075" dir="ltr">10.1k</span><span title="3 gold badges" aria-hidden="true"><span class="badge1"></span><span class="badgecount">3</span></span><span class="v-visible-sr">3 gold badges</span><span title="27 silver badges" aria-hidden="true"><span class="badge2"></span><span class="badgecount">27</span></span><span class="v-visible-sr">27 silver badges</span><span title="57 bronze badges" aria-hidden="true"><span class="badge3"></span><span class="badgecount">57</span></span><span class="v-visible-sr">57 bronze badges</span>
</div>
</div>
</div>
</div>
</div>
</div>
<span class="d-none">\$\endgroup\$</span>
</div>
<div class="js-post-notices post-layout--full">
</div>
<div class="post-layout--right">
<div id="comments-364940" class="comments js-comments-container bt bc-black-2 mt12 " data-post-id="364940" data-min-length="15">
<ul class="comments-list js-comments-list"
data-remaining-comments-count="0"
data-canpost="false"
data-cansee="true"
data-comments-unavailable="false"
data-addlink-disabled="true">
<li id="comment-879712" class="comment js-comment " data-comment-id="879712">
<div class="js-comment-actions comment-actions">
<div class="comment-score js-comment-edit-hide">
<span title="number of &#39;useful comment&#39; votes received"
class="warm">11</span>
</div>
</div>
<div class="comment-text js-comment-text-and-form">
<div class="comment-body js-comment-edit-hide">
<span class="d-none">\$\begingroup\$</span>
<span class="comment-copy">for a binary system, 2.048 is easier to use mathematically</span>
<span class="d-none">\$\endgroup\$</span>
&ndash;&nbsp;<a href="/users/17409/alnitak"
title="176 reputation"
class="comment-user">Alnitak</a>
<span class="comment-date" dir="ltr"><span title="2018-03-28 09:35:41Z" class="relativetime-clean">Mar 28 '18 at 9:35</span></span>
</div>
</div>
</li>
<li id="comment-880370" class="comment js-comment " data-comment-id="880370">
<div class="js-comment-actions comment-actions">
<div class="comment-score js-comment-edit-hide">
</div>
</div>
<div class="comment-text js-comment-text-and-form">
<div class="comment-body js-comment-edit-hide">
<span class="d-none">\$\begingroup\$</span>
<span class="comment-copy">@Aln: Not really. It just <i>looks</i> easier to the non-thinking at first glance.</span>
<span class="d-none">\$\endgroup\$</span>
&ndash;&nbsp;<a href="/users/4512/olin-lathrop"
title="290,382 reputation"
class="comment-user">Olin Lathrop</a>
<span class="comment-date" dir="ltr"><span title="2018-03-29 11:25:20Z" class="relativetime-clean">Mar 29 '18 at 11:25</span></span>
</div>
</div>
</li>
</ul>
</div>
<div id="comments-link-364940" data-rep=50 data-anon=true>
<a class="js-add-link comments-link disabled-link" title="Use comments to ask for more information or suggest improvements. Avoid answering questions in comments." >add a comment</a>
<span class="js-link-separator dno">&nbsp;|&nbsp;</span>
<a class="js-show-link comments-link dno" title="expand to show all comments on this post" href=# onclick=""></a>
</div>
</div> </div>
</div>
<div id="answers">
<a name="tab-top"></a>
<div id="answers-header">
<div class="subheader answers-subheader">
<h2 data-answercount="4">
4 Answers
<span style="display:none;" itemprop="answerCount">4</span>
</h2>
<div>
<div id="tabs">
<a href="/questions/364940/why-use-2-048v-and-4-096-as-a-reference?answertab=active#tab-top" data-nav-xhref="" title="Answers with the latest activity first" data-value="active" data-shortcut="A">
active</a>
<a href="/questions/364940/why-use-2-048v-and-4-096-as-a-reference?answertab=oldest#tab-top" data-nav-xhref="" title="Answers in the order they were provided" data-value="oldest" data-shortcut="O">
oldest</a>
<a class="youarehere is-selected " href="/questions/364940/why-use-2-048v-and-4-096-as-a-reference?answertab=votes#tab-top" data-nav-xhref="" title="Answers with the highest score first" data-value="votes" data-shortcut="V">
votes</a>
</div>
</div>
</div>
</div>
<a name="364941"></a>
<div id="answer-364941" class="answer accepted-answer" data-answerid="364941" itemprop="acceptedAnswer" itemscope itemtype="http://schema.org/Answer">
<div class="post-layout">
<div class="votecell post-layout--left">
<div class="js-voting-container grid fd-column ai-stretch gs4 fc-black-200" data-post-id="364941">
<button class="js-vote-up-btn grid--cell s-btn s-btn__unset c-pointer" title="This answer is useful" aria-pressed="false" aria-label="up vote" data-selected-classes="fc-theme-primary"><svg aria-hidden="true" class="svg-icon m0 iconArrowUpLg" width="36" height="36" viewBox="0 0 36 36"><path d="M2 26h32L18 10 2 26z"/></svg></button>
<div class="js-vote-count grid--cell fc-black-500 fs-title grid fd-column ai-center" itemprop="upvoteCount" data-value="49">49</div>
<button class="js-vote-down-btn grid--cell s-btn s-btn__unset c-pointer" title="This answer is not useful" aria-pressed="false" aria-label="down vote" data-selected-classes="fc-theme-primary"><svg aria-hidden="true" class="svg-icon m0 iconArrowDownLg" width="36" height="36" viewBox="0 0 36 36"><path d="M2 10h32L18 26 2 10z"/></svg></button>
<div class="js-accepted-answer-indicator grid--cell fc-green-500 ta-center py4" title="loading when this answer was accepted..." tabindex="0" role="note" aria-label="accepted">
<svg aria-hidden="true" class="svg-icon iconCheckmarkLg" width="36" height="36" viewBox="0 0 36 36"><path d="M6 14l8 8L30 6v8L14 30l-8-8v-8z"/></svg>
</div>
</div>
</div>
<div class="answercell post-layout--right">
<span class="d-none">\$\begingroup\$</span>
<div class="post-text" itemprop="text">
<p>When quantising voltages (i.e. passing through a ADC), you usually convert the voltage to an integer representation which is represented using a power of 2 scheme. </p>
<p>This means that they fall into the pattern of binary numbers, e.g. an 8 bit DAC has 256 individual levels. Using a reference that has a power of 2 number of millivolts means that the actual digital values have significant values.</p>
<p>For example, if you have an 11-bit DAC with a reference of 2.048, then the digital value is the number of millivolts.</p>
<p><strong>Edit</strong>: As pointed out by Andrew Morton, this provides 2048 levels, whereas there are 2049 millivolt levels including 0. So therefore to properly represent each bit as a millivolt you would require an extra bit. However if you round consistently, it is still possible to round each element down and achieve 0-2047 mV, or round up and have 1-2048 mV. If you fit 2048 to 2049 then you lose the nice property of directly matching the number of millivolts.</p>
</div>
<div class="grid mb0 fw-wrap ai-start jc-end gs8 gsy">
<time itemprop="dateCreated" datetime="2018-03-27T15:01:55"></time>
<div class="grid--cell mr16" style="flex: 1 1 100px;">
<div class="post-menu"><a href="/a/364941" itemprop="url"
class="js-share-link js-gps-track"
title="short permalink to this answer"
data-gps-track="post.click({ item: 2, priv: 0, post_type: 2 })"
data-controller="se-share-sheet"
data-se-share-sheet-title="Share a link to this answer"
data-se-share-sheet-subtitle=""
data-se-share-sheet-post-type="answer"
data-se-share-sheet-social="facebook twitter "
data-se-share-sheet-location="2"
data-s-popover-placement="bottom-start">share</a><span class="lsep">|</span><a href="/posts/364941/edit" class="suggest-edit-post js-gps-track" data-gps-track="post.click({ item: 6, priv: 0, post_type: 2 })" title="">improve this answer</a></div>
</div>
<div class="post-signature grid--cell fl0">
<div class="user-info ">
<div class="user-action-time">
<a href="/posts/364941/revisions" title="show all edits to this post"
class="js-gps-track"
data-gps-track="post.click({ item: 4, priv: 0, post_type: 2 })">edited <span title="2018-03-28 15:27:30Z" class="relativetime">Mar 28 '18 at 15:27</span></a>
</div>
<div class="user-gravatar32">
</div>
<div class="user-details">
<div class="-flair">
</div>
</div>
</div> </div>
<div class="post-signature grid--cell fl0">
<div class="user-info ">
<div class="user-action-time">
answered <span title="2018-03-27 15:01:55Z" class="relativetime">Mar 27 '18 at 15:01</span>
</div>
<div class="user-gravatar32">
<a href="/users/92505/loudnoises"><div class="gravatar-wrapper-32"><img src="https://i.stack.imgur.com/OU12r.jpg?s=32&amp;g=1" alt="" width="32" height="32" class="bar-sm"></div></a>
</div>
<div class="user-details" itemprop="author" itemscope itemtype="http://schema.org/Person">
<a href="/users/92505/loudnoises">loudnoises</a><span class="d-none" itemprop="name">loudnoises</span>
<div class="-flair">
<span class="reputation-score" title="reputation score " dir="ltr">1,666</span><span title="10 silver badges" aria-hidden="true"><span class="badge2"></span><span class="badgecount">10</span></span><span class="v-visible-sr">10 silver badges</span><span title="23 bronze badges" aria-hidden="true"><span class="badge3"></span><span class="badgecount">23</span></span><span class="v-visible-sr">23 bronze badges</span>
</div>
</div>
</div>
</div>
</div>
<span class="d-none">\$\endgroup\$</span>
</div>
<div class="js-post-notices post-layout--full">
</div>
<div class="post-layout--right">
<div id="comments-364941" class="comments js-comments-container bt bc-black-2 mt12 " data-post-id="364941" data-min-length="15">
<ul class="comments-list js-comments-list"
data-remaining-comments-count="0"
data-canpost="false"
data-cansee="true"
data-comments-unavailable="false"
data-addlink-disabled="true">
<li id="comment-879573" class="comment js-comment " data-comment-id="879573">
<div class="js-comment-actions comment-actions">
<div class="comment-score js-comment-edit-hide">
<span title="number of &#39;useful comment&#39; votes received"
class="cool">3</span>
</div>
</div>
<div class="comment-text js-comment-text-and-form">
<div class="comment-body js-comment-edit-hide">
<span class="d-none">\$\begingroup\$</span>
<span class="comment-copy">Which is very useful for metering using a comparator. One of the reasons we can buy a $5 multimeter.</span>
<span class="d-none">\$\endgroup\$</span>
&ndash;&nbsp;<a href="/users/56601/mckenzm"
title="147 reputation"
class="comment-user">mckenzm</a>
<span class="comment-date" dir="ltr"><span title="2018-03-28 02:09:01Z" class="relativetime-clean">Mar 28 '18 at 2:09</span></span>
<span title="this comment was edited 1 time">
<svg aria-hidden="true" class="svg-icon va-text-bottom o50 iconPencilSm" width="14" height="14" viewBox="0 0 14 14"><path d="M11.1 1.71l1.13 1.12c.2.2.2.51 0 .71L11.1 4.7 9.21 2.86l1.17-1.15c.2-.2.51-.2.71 0zM2 10.12l6.37-6.43 1.88 1.88L3.88 12H2v-1.88z"/></svg>
</span>
</div>
</div>
</li>
<li id="comment-879812" class="comment js-comment " data-comment-id="879812">
<div class="js-comment-actions comment-actions">
<div class="comment-score js-comment-edit-hide">
<span title="number of &#39;useful comment&#39; votes received"
class="cool">1</span>
</div>
</div>
<div class="comment-text js-comment-text-and-form">
<div class="comment-body js-comment-edit-hide">
<span class="d-none">\$\begingroup\$</span>
<span class="comment-copy">Ummm.... 11111111111 (binary) is 2047. Does that mean that an input of zero to the DAC will output 1 mV, or that an input of 1024 will output 1024.5 mV?</span>
<span class="d-none">\$\endgroup\$</span>
&ndash;&nbsp;<a href="/users/36731/andrew-morton"
title="1,512 reputation"
class="comment-user">Andrew Morton</a>
<span class="comment-date" dir="ltr"><span title="2018-03-28 13:46:55Z" class="relativetime-clean">Mar 28 '18 at 13:46</span></span>
</div>
</div>
</li>
<li id="comment-879815" class="comment js-comment " data-comment-id="879815">
<div class="js-comment-actions comment-actions">
<div class="comment-score js-comment-edit-hide">
</div>
</div>
<div class="comment-text js-comment-text-and-form">
<div class="comment-body js-comment-edit-hide">
<span class="d-none">\$\begingroup\$</span>
<span class="comment-copy">Ah good point! Well it matches the same number of levels, but with an off by one error.</span>
<span class="d-none">\$\endgroup\$</span>
&ndash;&nbsp;<a href="/users/92505/loudnoises"
title="1,666 reputation"
class="comment-user">loudnoises</a>
<span class="comment-date" dir="ltr"><span title="2018-03-28 13:53:43Z" class="relativetime-clean">Mar 28 '18 at 13:53</span></span>
</div>
</div>
</li>
<li id="comment-880278" class="comment js-comment " data-comment-id="880278">
<div class="js-comment-actions comment-actions">
<div class="comment-score js-comment-edit-hide">
</div>
</div>
<div class="comment-text js-comment-text-and-form">
<div class="comment-body js-comment-edit-hide">
<span class="d-none">\$\begingroup\$</span>
<span class="comment-copy">That&#39;s a fence-post error and it halves every time you increase the number of bits by one. (as does the quantization error).</span>
<span class="d-none">\$\endgroup\$</span>
&ndash;&nbsp;<a href="/users/25535/rodney"
title="126 reputation"
class="comment-user">Rodney</a>
<span class="comment-date" dir="ltr"><span title="2018-03-29 07:43:19Z" class="relativetime-clean">Mar 29 '18 at 7:43</span></span>
<span title="this comment was edited 1 time">
<svg aria-hidden="true" class="svg-icon va-text-bottom o50 iconPencilSm" width="14" height="14" viewBox="0 0 14 14"><path d="M11.1 1.71l1.13 1.12c.2.2.2.51 0 .71L11.1 4.7 9.21 2.86l1.17-1.15c.2-.2.51-.2.71 0zM2 10.12l6.37-6.43 1.88 1.88L3.88 12H2v-1.88z"/></svg>
</span>
</div>
</div>
</li>
<li id="comment-880359" class="comment js-comment " data-comment-id="880359">
<div class="js-comment-actions comment-actions">
<div class="comment-score js-comment-edit-hide">
</div>
</div>
<div class="comment-text js-comment-text-and-form">
<div class="comment-body js-comment-edit-hide">
<span class="d-none">\$\begingroup\$</span>
<span class="comment-copy">The typical bread-and-butter ADC actually seems to be 12bit, and if the high bit is used as a sign, and the voltage range is configured to be bipolar - centered around 0V, the 12bit signed integer actually has a range of -2048..+2047. Makes me wonder if the &quot;binary even&quot; reference could somehow be related to the negative end of the range by any chance :-)</span>
<span class="d-none">\$\endgroup\$</span>
&ndash;&nbsp;<a href="/users/94121/frr"
title="920 reputation"
class="comment-user">frr</a>
<span class="comment-date" dir="ltr"><span title="2018-03-29 11:05:36Z" class="relativetime-clean">Mar 29 '18 at 11:05</span></span>
</div>
</div>
</li>
</ul>
</div>
<div id="comments-link-364941" data-rep=50 data-anon=true>
<a class="js-add-link comments-link disabled-link" title="Use comments to ask for more information or suggest improvements. Avoid comments like “+1” or “thanks”." >add a comment</a>
<span class="js-link-separator dno">&nbsp;|&nbsp;</span>
<a class="js-show-link comments-link dno" title="expand to show all comments on this post" href=# onclick=""></a>
</div>
</div> </div>
</div>
<div id="dfp-mlb" class="everyonelovesstackoverflow everyoneloves__mid-leaderboard"></div>
<a name="364942"></a>
<div id="answer-364942" class="answer" data-answerid="364942" itemprop="suggestedAnswer" itemscope itemtype="http://schema.org/Answer">
<div class="post-layout">
<div class="votecell post-layout--left">
<div class="js-voting-container grid fd-column ai-stretch gs4 fc-black-200" data-post-id="364942">
<button class="js-vote-up-btn grid--cell s-btn s-btn__unset c-pointer" title="This answer is useful" aria-pressed="false" aria-label="up vote" data-selected-classes="fc-theme-primary"><svg aria-hidden="true" class="svg-icon m0 iconArrowUpLg" width="36" height="36" viewBox="0 0 36 36"><path d="M2 26h32L18 10 2 26z"/></svg></button>
<div class="js-vote-count grid--cell fc-black-500 fs-title grid fd-column ai-center" itemprop="upvoteCount" data-value="21">21</div>
<button class="js-vote-down-btn grid--cell s-btn s-btn__unset c-pointer" title="This answer is not useful" aria-pressed="false" aria-label="down vote" data-selected-classes="fc-theme-primary"><svg aria-hidden="true" class="svg-icon m0 iconArrowDownLg" width="36" height="36" viewBox="0 0 36 36"><path d="M2 10h32L18 26 2 10z"/></svg></button>
<div class="js-accepted-answer-indicator grid--cell fc-green-500 ta-center py4 d-none" title="loading when this answer was accepted..." tabindex="0" role="note" aria-label="accepted">
<svg aria-hidden="true" class="svg-icon iconCheckmarkLg" width="36" height="36" viewBox="0 0 36 36"><path d="M6 14l8 8L30 6v8L14 30l-8-8v-8z"/></svg>
</div>
</div>
</div>
<div class="answercell post-layout--right">
<span class="d-none">\$\begingroup\$</span>
<div class="post-text" itemprop="text">
<p>4.096V and 2.048V references allow the ADC to generate an integer value in mV. It means each step of the ADC represents 1mV or an integer multiple of 1mV.
4.096V = 2^12 mV </p>
</div>
<div class="grid mb0 fw-wrap ai-start jc-end gs8 gsy">
<time itemprop="dateCreated" datetime="2018-03-27T15:02:23"></time>
<div class="grid--cell mr16" style="flex: 1 1 100px;">
<div class="post-menu"><a href="/a/364942" itemprop="url"
class="js-share-link js-gps-track"
title="short permalink to this answer"
data-gps-track="post.click({ item: 2, priv: 0, post_type: 2 })"
data-controller="se-share-sheet"
data-se-share-sheet-title="Share a link to this answer"
data-se-share-sheet-subtitle=""
data-se-share-sheet-post-type="answer"
data-se-share-sheet-social="facebook twitter "
data-se-share-sheet-location="2"
data-s-popover-placement="bottom-start">share</a><span class="lsep">|</span><a href="/posts/364942/edit" class="suggest-edit-post js-gps-track" data-gps-track="post.click({ item: 6, priv: 0, post_type: 2 })" title="">improve this answer</a></div>
</div>
<div class="post-signature grid--cell fl0">
<div class="user-info user-hover">
<div class="user-action-time">
answered <span title="2018-03-27 15:02:23Z" class="relativetime">Mar 27 '18 at 15:02</span>
</div>
<div class="user-gravatar32">
<a href="/users/17849/lior-bilia"><div class="gravatar-wrapper-32"><img src="https://www.gravatar.com/avatar/633ca36c46e99b067400c2b01ad9fa65?s=32&amp;d=identicon&amp;r=PG" alt="" width="32" height="32" class="bar-sm"></div></a>
</div>
<div class="user-details" itemprop="author" itemscope itemtype="http://schema.org/Person">
<a href="/users/17849/lior-bilia">Lior Bilia</a><span class="d-none" itemprop="name">Lior Bilia</span>
<div class="-flair">
<span class="reputation-score" title="reputation score " dir="ltr">4,961</span><span title="1 gold badge" aria-hidden="true"><span class="badge1"></span><span class="badgecount">1</span></span><span class="v-visible-sr">1 gold badge</span><span title="12 silver badges" aria-hidden="true"><span class="badge2"></span><span class="badgecount">12</span></span><span class="v-visible-sr">12 silver badges</span><span title="24 bronze badges" aria-hidden="true"><span class="badge3"></span><span class="badgecount">24</span></span><span class="v-visible-sr">24 bronze badges</span>
</div>
</div>
</div>
</div>
</div>
<span class="d-none">\$\endgroup\$</span>
</div>
<div class="js-post-notices post-layout--full">
</div>
<div class="post-layout--right">
<div id="comments-364942" class="comments js-comments-container bt bc-black-2 mt12 " data-post-id="364942" data-min-length="15">
<ul class="comments-list js-comments-list"
data-remaining-comments-count="0"
data-canpost="false"
data-cansee="true"
data-comments-unavailable="false"
data-addlink-disabled="true">
<li id="comment-879776" class="comment js-comment " data-comment-id="879776">
<div class="js-comment-actions comment-actions">
<div class="comment-score js-comment-edit-hide">
<span title="number of &#39;useful comment&#39; votes received"
class="cool">2</span>
</div>
</div>
<div class="comment-text js-comment-text-and-form">
<div class="comment-body js-comment-edit-hide">
<span class="d-none">\$\begingroup\$</span>
<span class="comment-copy">allow a 12 or 11-bit ADC, to be precise.</span>
<span class="d-none">\$\endgroup\$</span>
&ndash;&nbsp;<a href="/users/97482/jcaron"
title="173 reputation"
class="comment-user">jcaron</a>
<span class="comment-date" dir="ltr"><span title="2018-03-28 12:16:47Z" class="relativetime-clean">Mar 28 '18 at 12:16</span></span>
</div>
</div>
</li>
<li id="comment-879787" class="comment js-comment " data-comment-id="879787">
<div class="js-comment-actions comment-actions">
<div class="comment-score js-comment-edit-hide">
<span title="number of &#39;useful comment&#39; votes received"
class="cool">1</span>
</div>
</div>
<div class="comment-text js-comment-text-and-form">
<div class="comment-body js-comment-edit-hide">
<span class="d-none">\$\begingroup\$</span>
<span class="comment-copy">Higher resolution ADCs also benefit from such arrangement, to be exact, the software driving these ADC can use integers to represent 0.5mV, 0.25mV and so on instead of fixed point variables.</span>
<span class="d-none">\$\endgroup\$</span>
&ndash;&nbsp;<a href="/users/17849/lior-bilia"
title="4,961 reputation"
class="comment-user">Lior Bilia</a>
<span class="comment-date" dir="ltr"><span title="2018-03-28 12:43:19Z" class="relativetime-clean">Mar 28 '18 at 12:43</span></span>
</div>
</div>
</li>
<li id="comment-879788" class="comment js-comment " data-comment-id="879788">
<div class="js-comment-actions comment-actions">
<div class="comment-score js-comment-edit-hide">
</div>
</div>
<div class="comment-text js-comment-text-and-form">
<div class="comment-body js-comment-edit-hide">
<span class="d-none">\$\begingroup\$</span>
<span class="comment-copy">Higher and lower resolutions ADCs definitely benefit from the arrangement, but to get specifically 1 mV steps as you state in your answer, you need to match the reference with the resolution.</span>
<span class="d-none">\$\endgroup\$</span>
&ndash;&nbsp;<a href="/users/97482/jcaron"
title="173 reputation"
class="comment-user">jcaron</a>
<span class="comment-date" dir="ltr"><span title="2018-03-28 12:45:12Z" class="relativetime-clean">Mar 28 '18 at 12:45</span></span>
</div>
</div>
</li>
<li id="comment-879796" class="comment js-comment " data-comment-id="879796">
<div class="js-comment-actions comment-actions">
<div class="comment-score js-comment-edit-hide">
</div>
</div>
<div class="comment-text js-comment-text-and-form">
<div class="comment-body js-comment-edit-hide">
<span class="d-none">\$\begingroup\$</span>
<span class="comment-copy">That is the ideal, yes, but sadly there is no 16.777216V reference out there.</span>
<span class="d-none">\$\endgroup\$</span>
&ndash;&nbsp;<a href="/users/17849/lior-bilia"
title="4,961 reputation"
class="comment-user">Lior Bilia</a>
<span class="comment-date" dir="ltr"><span title="2018-03-28 13:08:27Z" class="relativetime-clean">Mar 28 '18 at 13:08</span></span>
</div>
</div>
</li>
</ul>
</div>
<div id="comments-link-364942" data-rep=50 data-anon=true>
<a class="js-add-link comments-link disabled-link" title="Use comments to ask for more information or suggest improvements. Avoid comments like “+1” or “thanks”." >add a comment</a>
<span class="js-link-separator dno">&nbsp;|&nbsp;</span>
<a class="js-show-link comments-link dno" title="expand to show all comments on this post" href=# onclick=""></a>
</div>
</div> </div>
</div>
<a name="364946"></a>
<div id="answer-364946" class="answer" data-answerid="364946" itemprop="suggestedAnswer" itemscope itemtype="http://schema.org/Answer">
<div class="post-layout">
<div class="votecell post-layout--left">
<div class="js-voting-container grid fd-column ai-stretch gs4 fc-black-200" data-post-id="364946">
<button class="js-vote-up-btn grid--cell s-btn s-btn__unset c-pointer" title="This answer is useful" aria-pressed="false" aria-label="up vote" data-selected-classes="fc-theme-primary"><svg aria-hidden="true" class="svg-icon m0 iconArrowUpLg" width="36" height="36" viewBox="0 0 36 36"><path d="M2 26h32L18 10 2 26z"/></svg></button>
<div class="js-vote-count grid--cell fc-black-500 fs-title grid fd-column ai-center" itemprop="upvoteCount" data-value="10">10</div>
<button class="js-vote-down-btn grid--cell s-btn s-btn__unset c-pointer" title="This answer is not useful" aria-pressed="false" aria-label="down vote" data-selected-classes="fc-theme-primary"><svg aria-hidden="true" class="svg-icon m0 iconArrowDownLg" width="36" height="36" viewBox="0 0 36 36"><path d="M2 10h32L18 26 2 10z"/></svg></button>
<div class="js-accepted-answer-indicator grid--cell fc-green-500 ta-center py4 d-none" title="loading when this answer was accepted..." tabindex="0" role="note" aria-label="accepted">
<svg aria-hidden="true" class="svg-icon iconCheckmarkLg" width="36" height="36" viewBox="0 0 36 36"><path d="M6 14l8 8L30 6v8L14 30l-8-8v-8z"/></svg>
</div>
</div>
</div>
<div class="answercell post-layout--right">
<span class="d-none">\$\begingroup\$</span>
<div class="post-text" itemprop="text">
<p>The reason for these is because they can be easily divided down to a base 2. This makes them useful for things like ADCs where a 12-bit ADC with a rail between 0 and 4.096V will mean 1mV per bit, which is a much easier number.</p>
<p>There are also more voltages that do the same thing. You can also get Voltage references in 1.024V, which is 2<sup>10</sup>. Different references can be used for different bit ADCs.</p>
</div>
<div class="grid mb0 fw-wrap ai-start jc-end gs8 gsy">
<time itemprop="dateCreated" datetime="2018-03-27T15:18:55"></time>
<div class="grid--cell mr16" style="flex: 1 1 100px;">
<div class="post-menu"><a href="/a/364946" itemprop="url"
class="js-share-link js-gps-track"
title="short permalink to this answer"
data-gps-track="post.click({ item: 2, priv: 0, post_type: 2 })"
data-controller="se-share-sheet"
data-se-share-sheet-title="Share a link to this answer"
data-se-share-sheet-subtitle=""
data-se-share-sheet-post-type="answer"
data-se-share-sheet-social="facebook twitter "
data-se-share-sheet-location="2"
data-s-popover-placement="bottom-start">share</a><span class="lsep">|</span><a href="/posts/364946/edit" class="suggest-edit-post js-gps-track" data-gps-track="post.click({ item: 6, priv: 0, post_type: 2 })" title="">improve this answer</a></div>
</div>
<div class="post-signature grid--cell fl0">
<div class="user-info ">
<div class="user-action-time">
<a href="/posts/364946/revisions" title="show all edits to this post"
class="js-gps-track"
data-gps-track="post.click({ item: 4, priv: 0, post_type: 2 })">edited <span title="2018-03-29 10:22:03Z" class="relativetime">Mar 29 '18 at 10:22</span></a>
</div>
<div class="user-gravatar32">
<a href="/users/113522/glorfindel"><div class="gravatar-wrapper-32"><img src="https://i.stack.imgur.com/Haz6W.jpg?s=32&amp;g=1" alt="" width="32" height="32" class="bar-sm"></div></a>
</div>
<div class="user-details">
<a href="/users/113522/glorfindel">Glorfindel</a>
<div class="-flair">
<span class="reputation-score" title="reputation score " dir="ltr">695</span><span title="1 gold badge" aria-hidden="true"><span class="badge1"></span><span class="badgecount">1</span></span><span class="v-visible-sr">1 gold badge</span><span title="7 silver badges" aria-hidden="true"><span class="badge2"></span><span class="badgecount">7</span></span><span class="v-visible-sr">7 silver badges</span><span title="12 bronze badges" aria-hidden="true"><span class="badge3"></span><span class="badgecount">12</span></span><span class="v-visible-sr">12 bronze badges</span>
</div>
</div>
</div> </div>
<div class="post-signature grid--cell fl0">
<div class="user-info ">
<div class="user-action-time">
answered <span title="2018-03-27 15:18:55Z" class="relativetime">Mar 27 '18 at 15:18</span>
</div>
<div class="user-gravatar32">
<a href="/users/159450/curious"><div class="gravatar-wrapper-32"><img src="https://www.gravatar.com/avatar/026517fb7628b215c829306fdedfaf55?s=32&amp;d=identicon&amp;r=PG&amp;f=1" alt="" width="32" height="32" class="bar-sm"></div></a>
</div>
<div class="user-details" itemprop="author" itemscope itemtype="http://schema.org/Person">
<a href="/users/159450/curious">Curious</a><span class="d-none" itemprop="name">Curious</span>
<div class="-flair">
<span class="reputation-score" title="reputation score " dir="ltr">480</span><span title="5 silver badges" aria-hidden="true"><span class="badge2"></span><span class="badgecount">5</span></span><span class="v-visible-sr">5 silver badges</span><span title="17 bronze badges" aria-hidden="true"><span class="badge3"></span><span class="badgecount">17</span></span><span class="v-visible-sr">17 bronze badges</span>
</div>
</div>
</div>
</div>
</div>
<span class="d-none">\$\endgroup\$</span>
</div>
<div class="js-post-notices post-layout--full">
</div>
<div class="post-layout--right">
<div id="comments-364946" class="comments js-comments-container bt bc-black-2 mt12 dno" data-post-id="364946" data-min-length="15">
<ul class="comments-list js-comments-list"
data-remaining-comments-count="0"
data-canpost="false"
data-cansee="true"
data-comments-unavailable="false"
data-addlink-disabled="true">
</ul>
</div>
<div id="comments-link-364946" data-rep=50 data-anon=true>
<a class="js-add-link comments-link disabled-link" title="Use comments to ask for more information or suggest improvements. Avoid comments like “+1” or “thanks”." >add a comment</a>
<span class="js-link-separator dno">&nbsp;|&nbsp;</span>
<a class="js-show-link comments-link dno" title="expand to show all comments on this post" href=# onclick=""></a>
</div>
</div> </div>
</div>
<a name="365296"></a>
<div id="answer-365296" class="answer" data-answerid="365296" itemprop="suggestedAnswer" itemscope itemtype="http://schema.org/Answer">
<div class="post-layout">
<div class="votecell post-layout--left">
<div class="js-voting-container grid fd-column ai-stretch gs4 fc-black-200" data-post-id="365296">
<button class="js-vote-up-btn grid--cell s-btn s-btn__unset c-pointer" title="This answer is useful" aria-pressed="false" aria-label="up vote" data-selected-classes="fc-theme-primary"><svg aria-hidden="true" class="svg-icon m0 iconArrowUpLg" width="36" height="36" viewBox="0 0 36 36"><path d="M2 26h32L18 10 2 26z"/></svg></button>
<div class="js-vote-count grid--cell fc-black-500 fs-title grid fd-column ai-center" itemprop="upvoteCount" data-value="5">5</div>
<button class="js-vote-down-btn grid--cell s-btn s-btn__unset c-pointer" title="This answer is not useful" aria-pressed="false" aria-label="down vote" data-selected-classes="fc-theme-primary"><svg aria-hidden="true" class="svg-icon m0 iconArrowDownLg" width="36" height="36" viewBox="0 0 36 36"><path d="M2 10h32L18 26 2 10z"/></svg></button>
<div class="js-accepted-answer-indicator grid--cell fc-green-500 ta-center py4 d-none" title="loading when this answer was accepted..." tabindex="0" role="note" aria-label="accepted">
<svg aria-hidden="true" class="svg-icon iconCheckmarkLg" width="36" height="36" viewBox="0 0 36 36"><path d="M6 14l8 8L30 6v8L14 30l-8-8v-8z"/></svg>
</div>
</div>
</div>
<div class="answercell post-layout--right">
<span class="d-none">\$\begingroup\$</span>
<div class="post-text" itemprop="text">
<blockquote> Why do we use references at those voltages instead of just simply 2V and 4V</blockquote>
<p>This can be advantageous in just the right circumstances when the microcontroller is displaying values directly to a human. However, most of the time it's because there are lots of people out there that are bad at math or don't stop and actually think.</p>
<p>As others have already shown, 2.048 = 2<sup>11</sup>/1000 and 4.096 = 2<sup>12</sup>/1000. If you use a 12 bit A/D with a 4.096 V reference, each count is 1 mV.</p>
<p>However, stop and consider when that actually matters. There is nothing inherently special about units of millivolts. In terms of physics, they are a totally arbitrary unit for measuring EMF.</p>
<p>In a control system, for example, the units used for the various measured quantities can be anything you like, as long as you know what they are. If you are using fixed point, then you want the maximum value to nearly fill the number, and use enough bits so that you have the necessary resolution. The scaling of units should be dictated by convenient internal binary representations.</p>
<p>There will inevitably be adjustable gain factors later in the process anyway. Custom scaling of all the input values can be adjusted for by using different values of gain factors that are already there, and that the system already has to handle arbitrary values of. No additional computation is required, only different values fed into the same computations.</p>
<p>In some cases, these small embedded systems need to display digital values to humans. In that case, units of millivolts are useful when you want to show a voltage with three decimal places. However, human interfaces by their nature are slow compared to microcontrollers. Generally you don't want to update a digital display at more than 2 Hz. Converting a number to decimal digits already requires some arithmetic anyway. Scaling some internal value to match the displayed resolution is a rather minor additional step relative to that process.</p>
<p>Then also consider how often you actually want to measure a voltage in the range of 0 to 4.095 V, or at least most of that range. If you want to measure 0 to 5 V, then the 4.096 reference really doesn't help. You need to attenuate the signal into the A/D anyway, so reading the attenuated signal in units of millivolts confers no special advantage, even when displaying digital values.</p>
<p>So in short, in today's world with microcontrollers handling A/D readings, 2.048 and 4.096 V references mostly cater to a <i>perceived</i> need, and to knee-jerkers who don't think about the problem properly.</p>
</div>
<div class="grid mb0 fw-wrap ai-start jc-end gs8 gsy">
<time itemprop="dateCreated" datetime="2018-03-29T11:22:28"></time>
<div class="grid--cell mr16" style="flex: 1 1 100px;">
<div class="post-menu"><a href="/a/365296" itemprop="url"
class="js-share-link js-gps-track"
title="short permalink to this answer"
data-gps-track="post.click({ item: 2, priv: 0, post_type: 2 })"
data-controller="se-share-sheet"
data-se-share-sheet-title="Share a link to this answer"
data-se-share-sheet-subtitle=""
data-se-share-sheet-post-type="answer"
data-se-share-sheet-social="facebook twitter "
data-se-share-sheet-location="2"
data-s-popover-placement="bottom-start">share</a><span class="lsep">|</span><a href="/posts/365296/edit" class="suggest-edit-post js-gps-track" data-gps-track="post.click({ item: 6, priv: 0, post_type: 2 })" title="">improve this answer</a></div>
</div>
<div class="post-signature grid--cell fl0">
<div class="user-info user-hover">
<div class="user-action-time">
answered <span title="2018-03-29 11:22:28Z" class="relativetime">Mar 29 '18 at 11:22</span>
</div>
<div class="user-gravatar32">
<a href="/users/4512/olin-lathrop"><div class="gravatar-wrapper-32"><img src="https://www.gravatar.com/avatar/75cac6f65c17c8eb960d3563c674695e?s=32&amp;d=identicon&amp;r=PG" alt="" width="32" height="32" class="bar-sm"></div></a>
</div>
<div class="user-details" itemprop="author" itemscope itemtype="http://schema.org/Person">
<a href="/users/4512/olin-lathrop">Olin Lathrop</a><span class="d-none" itemprop="name">Olin Lathrop</span>
<div class="-flair">
<span class="reputation-score" title="reputation score 290,382" dir="ltr">290k</span><span title="32 gold badges" aria-hidden="true"><span class="badge1"></span><span class="badgecount">32</span></span><span class="v-visible-sr">32 gold badges</span><span title="362 silver badges" aria-hidden="true"><span class="badge2"></span><span class="badgecount">362</span></span><span class="v-visible-sr">362 silver badges</span><span title="815 bronze badges" aria-hidden="true"><span class="badge3"></span><span class="badgecount">815</span></span><span class="v-visible-sr">815 bronze badges</span>
</div>
</div>
</div>
</div>
</div>
<span class="d-none">\$\endgroup\$</span>
</div>
<div class="js-post-notices post-layout--full">
</div>
<div class="post-layout--right">
<div id="comments-365296" class="comments js-comments-container bt bc-black-2 mt12 " data-post-id="365296" data-min-length="15">
<ul class="comments-list js-comments-list"
data-remaining-comments-count="0"
data-canpost="false"
data-cansee="true"
data-comments-unavailable="false"
data-addlink-disabled="true">
<li id="comment-880458" class="comment js-comment " data-comment-id="880458">
<div class="js-comment-actions comment-actions">
<div class="comment-score js-comment-edit-hide">
</div>
</div>
<div class="comment-text js-comment-text-and-form">
<div class="comment-body js-comment-edit-hide">
<span class="d-none">\$\begingroup\$</span>
<span class="comment-copy">not everyone what needs a division of 1V (or portion thereof) is using a micro-controller</span>
<span class="d-none">\$\endgroup\$</span>
&ndash;&nbsp;<a href="/users/17409/alnitak"
title="176 reputation"
class="comment-user">Alnitak</a>
<span class="comment-date" dir="ltr"><span title="2018-03-29 14:41:41Z" class="relativetime-clean">Mar 29 '18 at 14:41</span></span>
</div>
</div>
</li>
<li id="comment-880680" class="comment js-comment " data-comment-id="880680">
<div class="js-comment-actions comment-actions">
<div class="comment-score js-comment-edit-hide">
</div>
</div>
<div class="comment-text js-comment-text-and-form">
<div class="comment-body js-comment-edit-hide">
<span class="d-none">\$\begingroup\$</span>
<span class="comment-copy">vote up and there is one thing you forgot to mention - I think - all discussed answers ignore, that 1024 ~= 1000 &lt;3% error and smaller the 3% is still expensive and hard to get (if tou keep the whole system in mind)</span>
<span class="d-none">\$\endgroup\$</span>
&ndash;&nbsp;<a href="/users/164961/halfbit"
title="101 reputation"
class="comment-user">halfbit</a>
<span class="comment-date" dir="ltr"><span title="2018-03-29 20:28:02Z" class="relativetime-clean">Mar 29 '18 at 20:28</span></span>
<span title="this comment was edited 1 time">
<svg aria-hidden="true" class="svg-icon va-text-bottom o50 iconPencilSm" width="14" height="14" viewBox="0 0 14 14"><path d="M11.1 1.71l1.13 1.12c.2.2.2.51 0 .71L11.1 4.7 9.21 2.86l1.17-1.15c.2-.2.51-.2.71 0zM2 10.12l6.37-6.43 1.88 1.88L3.88 12H2v-1.88z"/></svg>
</span>
</div>
</div>
</li>
</ul>
</div>
<div id="comments-link-365296" data-rep=50 data-anon=true>
<a class="js-add-link comments-link disabled-link" title="Use comments to ask for more information or suggest improvements. Avoid comments like “+1” or “thanks”." >add a comment</a>
<span class="js-link-separator dno">&nbsp;|&nbsp;</span>
<a class="js-show-link comments-link dno" title="expand to show all comments on this post" href=# onclick=""></a>
</div>
</div> </div>
</div>
<a name='new-answer'></a>
<form id="post-form" action="/questions/364940/answer/submit" method="post" class="js-add-answer-component post-form">
<input type="hidden" id="post-id" value="364940" />
<input type="hidden" id="qualityBanWarningShown" name="qualityBanWarningShown" value="false" />
<input type="hidden" name="referrer" value="" />
<h2 class="space">
Your Answer
</h2>
<script>
StackExchange.ifUsing("editor", function () {
return StackExchange.using("schematics", function () {
StackExchange.schematics.init();
});
}, "cicuitlab");
</script>
<script>
StackExchange.ready(function() {
var channelOptions = {
tags: "".split(" "),
id: "135"
};
initTagRenderer("".split(" "), "".split(" "), channelOptions);
StackExchange.using("externalEditor", function() {
// Have to fire editor after snippets, if snippets enabled
if (StackExchange.settings.snippets.snippetsEnabled) {
StackExchange.using("snippets", function() {
createEditor();
});
}
else {
createEditor();
}
});
function createEditor() {
StackExchange.prepareEditor({
heartbeatType: 'answer',
autoActivateHeartbeat: false,
convertImagesToLinks: false,
noModals: true,
showLowRepImageUploadWarning: true,
reputationToPostImages: null,
bindNavPrevention: true,
postfix: "",
imageUploader: {
brandingHtml: "Powered by \u003ca class=\"icon-imgur-white\" href=\"https://imgur.com/\"\u003e\u003c/a\u003e",
contentPolicyHtml: "User contributions licensed under \u003ca href=\"https://creativecommons.org/licenses/by-sa/4.0/\"\u003ecc by-sa 4.0 with attribution required\u003c/a\u003e \u003ca href=\"https://stackoverflow.com/legal/content-policy\"\u003e(content policy)\u003c/a\u003e",
allowUrls: true
},
onDemand: true,
discardSelector: ".discard-answer"
,immediatelyShowMarkdownHelp:true
});
}
});
</script>
<div id="post-editor" class="post-editor js-post-editor js-wz-element" data-wz-state = "8,16,32,64,128,256">
<div class="ps-relative">
<div class="wmd-container mb8">
<div id="wmd-button-bar" class="wmd-button-bar btr-sm"></div>
<div class="js-stacks-validation">
<div class="ps-relative">
<textarea id="wmd-input"
name="post-text"
class="wmd-input js-wz-element s-input bar0 js-post-body-field"
data-post-type-id="2"
cols="92" rows="15"
tabindex="101"
data-wz-state = "256"
data-min-length=""></textarea>
</div>
<div class="s-input-message mt4 d-none js-stacks-validation-message"></div>
</div>
</div>
</div>
<aside class="grid ai-start jc-space-between js-answer-help s-notice s-notice__warning pb0 pr4 pt4 mb8 d-none" role="status" aria-hidden="true">
<div class="grid--cell pt8">
<p>Thanks for contributing an answer to Electrical Engineering Stack Exchange!</p><ul><li>Please be sure to <em>answer the question</em>. Provide details and share your research!</li></ul><p>But <em>avoid</em> …</p><ul><li>Asking for help, clarification, or responding to other answers.</li><li>Making statements based on opinion; back them up with references or personal experience.</li></ul><p>Use MathJax to format equations. <a href="http://www.math.harvard.edu/texman/">MathJax reference</a>.</p><p>To learn more, see our <a href="/help/how-to-answer">tips on writing great answers</a>.</p>
</div>
<button class="grid--cell js-answer-help-close-btn s-btn s-btn__muted fc-dark">
<svg aria-hidden="true" class="svg-icon iconClear" width="18" height="18" viewBox="0 0 18 18"><path d="M15 4.41L13.59 3 9 7.59 4.41 3 3 4.41 7.59 9 3 13.59 4.41 15 9 10.41 13.59 15 15 13.59 10.41 9 15 4.41z"/></svg>
</button>
</aside>
<div id="draft-saved" class="draft-saved community-option fl" style="height:24px; display:none;">draft saved</div>
<div id="draft-discarded" class="draft-discarded community-option fl" style="height:24px; display:none;">draft discarded</div>
<div id="wmd-preview" class="wmd-preview" ></div>
<div></div>
<div class="edit-block">
<input id="fkey" name="fkey" type="hidden" value="6067184cd28298c9cd1163745c0315d13a2a3e1c0e8abc4ae1db30773cc2fc93">
<input id="author" name="author" type="text">
</div>
</div>
<div class="ps-relative">
<div class="form-item dno new-post-login p0 my16">
<div class="grid gs16 md:fd-column new-login-form">
<div class="grid fd-column w50 md:w-auto gsy gs8 jc-space-between new-login-left">
<h3 class="grid--cell fs-title">Sign up or <a id="login-link" href="/users/login?ssrc=question_page&returnurl=https%3a%2f%2felectronics.stackexchange.com%2fquestions%2f364940%2fwhy-use-2-048v-and-4-096-as-a-reference%23new-answer">log in</a></h3>
<script>
StackExchange.ready(function () {
StackExchange.helpers.onClickDraftSave('#login-link');
});
</script>
<div class="grid--cell s-btn s-btn__muted s-btn__outlined s-btn__icon google-login" data-ga="[&quot;sign up&quot;,&quot;Sign Up Started - Google&quot;,&quot;New Post&quot;,null,null]">
<svg aria-hidden="true" class="svg-icon native iconGoogle" width="18" height="18" viewBox="0 0 18 18"><path d="M16.51 8H8.98v3h4.3c-.18 1-.74 1.48-1.6 2.04v2.01h2.6a7.8 7.8 0 0 0 2.38-5.88c0-.57-.05-.66-.15-1.18z" fill="#4285F4"/><path d="M8.98 17c2.16 0 3.97-.72 5.3-1.94l-2.6-2a4.8 4.8 0 0 1-7.18-2.54H1.83v2.07A8 8 0 0 0 8.98 17z" fill="#34A853"/><path d="M4.5 10.52a4.8 4.8 0 0 1 0-3.04V5.41H1.83a8 8 0 0 0 0 7.18l2.67-2.07z" fill="#FBBC05"/><path d="M8.98 4.18c1.17 0 2.23.4 3.06 1.2l2.3-2.3A8 8 0 0 0 1.83 5.4L4.5 7.49a4.77 4.77 0 0 1 4.48-3.3z" fill="#EA4335"/></svg> Sign up using Google
</div>
<div class="grid--cell s-btn s-btn__muted s-btn__icon facebook-login" data-ga="[&quot;sign up&quot;,&quot;Sign Up Started - Facebook&quot;,&quot;New Post&quot;,null,null]">
<svg aria-hidden="true" class="svg-icon iconFacebook" width="18" height="18" viewBox="0 0 18 18"><path d="M3 1a2 2 0 0 0-2 2v12c0 1.1.9 2 2 2h12a2 2 0 0 0 2-2V3a2 2 0 0 0-2-2H3zm6.55 16v-6.2H7.46V8.4h2.09V6.61c0-2.07 1.26-3.2 3.1-3.2.88 0 1.64.07 1.87.1v2.16h-1.29c-1 0-1.19.48-1.19 1.18V8.4h2.39l-.31 2.42h-2.08V17h-2.5z" fill="#4167B2"/></svg> Sign up using Facebook
</div>
<div class="grid--cell s-btn s-btn__muted s-btn__outlined s-btn__icon stackexchange-login" data-ga="[&quot;sign up&quot;,&quot;Sign Up Navigation&quot;,&quot;New Post&quot;,null,null]">
<svg aria-hidden="true" class="svg-icon native iconLogoGlyphXSm" width="18" height="18" viewBox="0 0 18 18"><path d="M14 16v-5h2v7H2v-7h2v5h10z" fill="#BCBBBB"/><path d="M12.09.72l-1.21.9 4.5 6.07 1.22-.9L12.09.71zM5 15h8v-2H5v2zm9.15-5.87L8.35 4.3l.96-1.16 5.8 4.83-.96 1.16zm-7.7-1.47l6.85 3.19.63-1.37-6.85-3.2-.63 1.38zm6.53 5L5.4 11.39l.38-1.67 7.42 1.48-.22 1.46z" fill="#F48024"/></svg> Sign up using Email and Password
</div>
</div>
<input type="hidden" name="use-facebook" class="use-facebook" value="false" />
<input type="hidden" name="use-google" class="use-google" value="false" />
<input type="button" class="submit-openid" value="Submit" style="display:none" />
<div class="grid gsy gs8 fd-column w50 md:w-auto new-login-right form-item p0">
<h3 class="grid--cell fs-title">Post as a guest</h3>
<div class="grid--cell">
<div class="grid gs4 gsy fd-column">
<label class="s-label" for="display-name">Name</label>
<div class="grid ps-relative">
<input class="s-input" id="display-name" name="display-name" maxlength="30" type="text" value="" tabindex="105" placeholder="" />
</div>
</div>
</div>
<div class="grid--cell">
<div class="grid gs4 gsy fd-column">
<div class="grid--cell">
<div class="grid gs2 gsy fd-column">
<label class="grid--cell s-label" for="m-address">Email</label>
<p class="grid--cell s-description">Required, but never shown</p>
</div>
</div>
<div class="grid ps-relative">
<input class="s-input js-post-email-field" id="m-address" name="m-address" type="text" value="" size="40" tabindex="106" placeholder="" />
</div>
</div>
</div>
</div>
</div>
</div>
<script>
StackExchange.ready(
function () {
StackExchange.openid.initPostLogin('.new-post-login', 'https%3a%2f%2felectronics.stackexchange.com%2fquestions%2f364940%2fwhy-use-2-048v-and-4-096-as-a-reference%23new-answer', 'question_page');
}
);
</script>
<noscript>
<h3 class="grid--cell fs-title">Post as a guest</h3>
<div class="grid--cell">
<div class="grid gs4 gsy fd-column">
<label class="s-label" for="display-name">Name</label>
<div class="grid ps-relative">
<input class="s-input" id="display-name" name="display-name" maxlength="30" type="text" value="" tabindex="105" placeholder="" />
</div>
</div>
</div>
<div class="grid--cell">
<div class="grid gs4 gsy fd-column">
<div class="grid--cell">
<div class="grid gs2 gsy fd-column">
<label class="grid--cell s-label" for="m-address">Email</label>
<p class="grid--cell s-description">Required, but never shown</p>
</div>
</div>
<div class="grid ps-relative">
<input class="s-input js-post-email-field" id="m-address" name="m-address" type="text" value="" size="40" tabindex="106" placeholder="" />
</div>
</div>
</div>
</noscript>
</div>
<div class="form-submit cbt grid gsx gs4">
<button id="submit-button" class="grid--cell s-btn s-btn__primary s-btn__icon" type="submit" tabindex="120" autocomplete="off">
Post Your Answer </button>
<button class="grid--cell s-btn s-btn__danger discard-answer dno">
Discard
</button>
<p class="privacy-policy-agreement">
By clicking “Post Your Answer”, you agree to our <a href='https://stackoverflow.com/legal/terms-of-service/public' name='tos' target='_blank' class='-link'>terms of service</a>, <a href='https://stackoverflow.com/legal/privacy-policy' name='privacy' target='_blank' class='-link'>privacy policy</a> and <a href='https://stackoverflow.com/legal/cookie-policy' name='cookie' target='_blank' class='-link'>cookie policy</a><input type="hidden" name="legalLinksShown" value="1" />
</p>
</div>
<div class="js-general-error general-error cbt d-none"></div>
</form>
<h2 class="bottom-notice" data-loc="1">
Not the answer you&#39;re looking for? Browse other questions tagged <a href="/questions/tagged/voltage-reference" class="post-tag" title="show questions tagged &#39;voltage-reference&#39;" rel="tag">voltage-reference</a> or <a href="/questions/ask">ask your own question</a>. </h2>
</div>
</div>
<div id="sidebar" class="show-votes" role="complementary" aria-label="sidebar">
<div class="s-sidebarwidget s-sidebarwidget__yellow s-anchors s-anchors__default mb16" data-tracker="cb=1">
<div class="s-sidebarwidget--header s-sidebarwidget__small-bold-text">
Blog
</div>
<ul class="s-sidebarwidget--content s-sidebarwidget__items pt0 pr16 pb0 pl16">
<li class="s-sidebarwidget--item grid">
<div class="grid--cell1 fl-shrink0">
<a href="https://stackoverflow.blog/2019/11/25/podcast-tfw-you-accidentally-delete-your-database/" data-ga="[&quot;community bulletin board&quot;,&quot;Blog&quot;,&quot;https://stackoverflow.blog/2019/11/25/podcast-tfw-you-accidentally-delete-your-database/&quot;,null,null]">
<div class="favicon favicon-stackexchangemeta" title="Meta Stack Exchange"></div> </a>
</div>
<div class="grid--cell">
<a href="https://stackoverflow.blog/2019/11/25/podcast-tfw-you-accidentally-delete-your-database/" class="js-gps-track" data-ga="[&quot;community bulletin board&quot;,&quot;Blog&quot;,&quot;https://stackoverflow.blog/2019/11/25/podcast-tfw-you-accidentally-delete-your-database/&quot;,null,null]" data-gps-track="communitybulletin.click({ priority: 1, position: 0 })">Podcast: TFW You Accidentally Delete Your Database</a>
</div>
</li>
</ul>
<ul class="s-sidebarwidget--content s-sidebarwidget__items pt0 pr16 pb0 pl16">
<li class="s-sidebarwidget--item grid">
<div class="grid--cell1 fl-shrink0">
<a href="https://stackoverflow.blog/2019/11/26/copying-code-from-stack-overflow-you-might-be-spreading-security-vulnerabilities/" data-ga="[&quot;community bulletin board&quot;,&quot;Blog&quot;,&quot;https://stackoverflow.blog/2019/11/26/copying-code-from-stack-overflow-you-might-be-spreading-security-vulnerabilities/&quot;,null,null]">
<div class="favicon favicon-stackexchangemeta" title="Meta Stack Exchange"></div> </a>
</div>
<div class="grid--cell">
<a href="https://stackoverflow.blog/2019/11/26/copying-code-from-stack-overflow-you-might-be-spreading-security-vulnerabilities/" class="js-gps-track" data-ga="[&quot;community bulletin board&quot;,&quot;Blog&quot;,&quot;https://stackoverflow.blog/2019/11/26/copying-code-from-stack-overflow-you-might-be-spreading-security-vulnerabilities/&quot;,null,null]" data-gps-track="communitybulletin.click({ priority: 1, position: 1 })">Copying code from Stack Overflow? You might paste security vulnerabilities, too</a>
</div>
</li>
</ul>
<div class="s-sidebarwidget--header s-sidebarwidget__small-bold-text">
Featured on Meta
</div>
<ul class="s-sidebarwidget--content s-sidebarwidget__items pt0 pr16 pb0 pl16">
<li class="s-sidebarwidget--item grid">
<div class="grid--cell1 fl-shrink0">
<a href="https://meta.stackexchange.com/questions/336177/feedback-post-moderator-review-and-reinstatement-processes" data-ga="[&quot;community bulletin board&quot;,&quot;Featured on Meta&quot;,&quot;https://meta.stackexchange.com/questions/336177/feedback-post-moderator-review-and-reinstatement-processes&quot;,null,null]">
<div class="favicon favicon-stackexchangemeta" title="Meta Stack Exchange"></div> </a>
</div>
<div class="grid--cell">
<a href="https://meta.stackexchange.com/questions/336177/feedback-post-moderator-review-and-reinstatement-processes" class="js-gps-track" data-ga="[&quot;community bulletin board&quot;,&quot;Featured on Meta&quot;,&quot;https://meta.stackexchange.com/questions/336177/feedback-post-moderator-review-and-reinstatement-processes&quot;,null,null]" data-gps-track="communitybulletin.click({ priority: 3, position: 2 })">Feedback post: Moderator review and reinstatement processes</a>
</div>
</li>
</ul>
</div>
<div id="dfp-tsb" class="everyonelovesstackoverflow everyoneloves__top-sidebar"></div>
<div class="module sidebar-related">
<h4 id="h-related">Related</h4>
<div class="related js-gps-related-questions" data-tracker="rq=1">
<div class="spacer"><a href="/q/9766" title="Vote score (upvotes - downvotes)"><div class="answer-votes default">2</div></a><a href="/questions/9766/what-half-bandgap-voltage-reference-is" class="question-hyperlink">What half-bandgap voltage reference is?</a></div><div class="spacer"><a href="/q/134360" title="Vote score (upvotes - downvotes)"><div class="answer-votes answered-accepted default">16</div></a><a href="/questions/134360/why-is-there-a-0r-resistor-linking-gnd-and-agnd-in-analog-voltage-reference-circ" class="question-hyperlink">Why is there a 0R resistor linking GND and AGND in analog voltage reference circuit?</a></div><div class="spacer"><a href="/q/164478" title="Vote score (upvotes - downvotes)"><div class="answer-votes answered-accepted default">0</div></a><a href="/questions/164478/use-reference-voltage-as-power-suppy" class="question-hyperlink">Use reference voltage as power suppy</a></div><div class="spacer"><a href="/q/246289" title="Vote score (upvotes - downvotes)"><div class="answer-votes default">0</div></a><a href="/questions/246289/precision-voltage-reference-for-multimeter" class="question-hyperlink">precision voltage reference for multimeter</a></div><div class="spacer"><a href="/q/345705" title="Vote score (upvotes - downvotes)"><div class="answer-votes default">0</div></a><a href="/questions/345705/how-do-you-generate-a-reference-voltage-in-a-circuit" class="question-hyperlink">How do you generate a reference voltage in a circuit?</a></div><div class="spacer"><a href="/q/353313" title="Vote score (upvotes - downvotes)"><div class="answer-votes default">1</div></a><a href="/questions/353313/resistor-between-voltage-reference-and-input-of-op-amp-why" class="question-hyperlink">Resistor Between Voltage Reference and + Input of Op-Amp? Why?</a></div><div class="spacer"><a href="/q/375210" title="Vote score (upvotes - downvotes)"><div class="answer-votes default">1</div></a><a href="/questions/375210/build-2v-reference-with-100ma-output" class="question-hyperlink">build 2V reference with 100mA output</a></div><div class="spacer"><a href="/q/383435" title="Vote score (upvotes - downvotes)"><div class="answer-votes default">2</div></a><a href="/questions/383435/using-positive-voltage-reference-on-a-negative-supply" class="question-hyperlink">Using positive voltage reference on a negative supply</a></div><div class="spacer"><a href="/q/466537" title="Vote score (upvotes - downvotes)"><div class="answer-votes answered-accepted default">0</div></a><a href="/questions/466537/mcp4725-dac-and-reference-volatge" class="question-hyperlink">MCP4725 DAC and reference volatge</a></div>
</div>
</div>
<div id="hot-network-questions" class="module tex2jax_ignore">
<h4>
<a href="https://stackexchange.com/questions?tab=hot"
class="js-gps-track s-link s-link__inherit"
data-gps-track="posts_hot_network.click({ item_type:1, location:11 })">
Hot Network Questions
</a>
</h4>
<ul>
<li >
<div class="favicon favicon-music" title="Music: Practice &amp; Theory Stack Exchange"></div><a href="https://music.stackexchange.com/questions/92425/anacruses-bar-lines-and-line-breaks" class="js-gps-track question-hyperlink mb0" data-gps-track="site.switch({ item_type:11, target_site:240 }); posts_hot_network.click({ item_type:2, location:11 })">
Anacruses, Bar Lines, and Line Breaks
</a>
</li>
<li >
<div class="favicon favicon-law" title="Law Stack Exchange"></div><a href="https://law.stackexchange.com/questions/46811/is-the-worst-version-of-the-accusations-against-president-trump-impeachable" class="js-gps-track question-hyperlink mb0" data-gps-track="site.switch({ item_type:11, target_site:617 }); posts_hot_network.click({ item_type:2, location:11 })">
Is the worst version of the accusations against President Trump impeachable?
</a>
</li>
<li >
<div class="favicon favicon-boardgames" title="Board &amp; Card Games Stack Exchange"></div><a href="https://boardgames.stackexchange.com/questions/49373/lazav-ability-on-the-stack" class="js-gps-track question-hyperlink mb0" data-gps-track="site.switch({ item_type:11, target_site:147 }); posts_hot_network.click({ item_type:2, location:11 })">
Lazav Ability on the Stack
</a>
</li>
<li >
<div class="favicon favicon-salesforce" title="Salesforce Stack Exchange"></div><a href="https://salesforce.stackexchange.com/questions/286463/queuable-apex-logic-in-constructor-or-execute-method" class="js-gps-track question-hyperlink mb0" data-gps-track="site.switch({ item_type:11, target_site:459 }); posts_hot_network.click({ item_type:2, location:11 })">
Queuable Apex logic in Constructor or Execute Method
</a>
</li>
<li >
<div class="favicon favicon-writing" title="Writing Stack Exchange"></div><a href="https://writing.stackexchange.com/questions/49065/in-academic-writing-why-do-some-recommend-to-avoid-announcing-the-topic" class="js-gps-track question-hyperlink mb0" data-gps-track="site.switch({ item_type:11, target_site:166 }); posts_hot_network.click({ item_type:2, location:11 })">
In academic writing why do some recommend to avoid &quot;announcing&quot; the topic?
</a>
</li>
<li class="dno js-hidden">
<div class="favicon favicon-unix" title="Unix &amp; Linux Stack Exchange"></div><a href="https://unix.stackexchange.com/questions/554476/extract-nth-pattern-from-a-file" class="js-gps-track question-hyperlink mb0" data-gps-track="site.switch({ item_type:11, target_site:106 }); posts_hot_network.click({ item_type:2, location:11 })">
extract nth pattern from a file
</a>
</li>
<li class="dno js-hidden">
<div class="favicon favicon-travel" title="Travel Stack Exchange"></div><a href="https://travel.stackexchange.com/questions/150168/why-are-compartments-in-western-european-day-trains-falling-out-of-fashion" class="js-gps-track question-hyperlink mb0" data-gps-track="site.switch({ item_type:11, target_site:273 }); posts_hot_network.click({ item_type:2, location:11 })">
Why are compartments in western European day trains falling out of fashion?
</a>
</li>
<li class="dno js-hidden">
<div class="favicon favicon-scifi" title="Science Fiction &amp; Fantasy Stack Exchange"></div><a href="https://scifi.stackexchange.com/questions/223674/are-there-indications-of-a-loss-of-past-historical-records-in-star-trek-universe" class="js-gps-track question-hyperlink mb0" data-gps-track="site.switch({ item_type:11, target_site:186 }); posts_hot_network.click({ item_type:2, location:11 })">
Are there indications of a loss of past historical records in Star Trek universe?
</a>
</li>
<li class="dno js-hidden">
<div class="favicon favicon-workplace" title="The Workplace Stack Exchange"></div><a href="https://workplace.stackexchange.com/questions/148719/how-can-i-deal-with-my-coworkers-using-unknown-jargon-and-acronyms" class="js-gps-track question-hyperlink mb0" data-gps-track="site.switch({ item_type:11, target_site:423 }); posts_hot_network.click({ item_type:2, location:11 })">
How can I deal with my coworkers using unknown jargon and acronyms?
</a>
</li>
<li class="dno js-hidden">
<div class="favicon favicon-askubuntu" title="Ask Ubuntu"></div><a href="https://askubuntu.com/questions/1191862/what-is-the-difference-between-more-and-less-commands" class="js-gps-track question-hyperlink mb0" data-gps-track="site.switch({ item_type:11, target_site:89 }); posts_hot_network.click({ item_type:2, location:11 })">
What is the difference between &quot;more&quot; and &quot;less&quot; commands?
</a>
</li>
<li class="dno js-hidden">
<div class="favicon favicon-workplace" title="The Workplace Stack Exchange"></div><a href="https://workplace.stackexchange.com/questions/148731/is-it-normal-to-not-be-able-to-work-8-hours-a-day" class="js-gps-track question-hyperlink mb0" data-gps-track="site.switch({ item_type:11, target_site:423 }); posts_hot_network.click({ item_type:2, location:11 })">
Is it normal to not be able to work 8 hours a day?
</a>
</li>
<li class="dno js-hidden">
<div class="favicon favicon-movies" title="Movies &amp; TV Stack Exchange"></div><a href="https://movies.stackexchange.com/questions/105294/what-did-rex-kramer-mean-by-routing-the-plane-in-lake-michigan" class="js-gps-track question-hyperlink mb0" data-gps-track="site.switch({ item_type:11, target_site:367 }); posts_hot_network.click({ item_type:2, location:11 })">
What did Rex Kramer mean by routing the plane in Lake Michigan?
</a>
</li>
<li class="dno js-hidden">
<div class="favicon favicon-puzzling" title="Puzzling Stack Exchange"></div><a href="https://puzzling.stackexchange.com/questions/91586/name-the-product" class="js-gps-track question-hyperlink mb0" data-gps-track="site.switch({ item_type:11, target_site:559 }); posts_hot_network.click({ item_type:2, location:11 })">
Name the product
</a>
</li>
<li class="dno js-hidden">
<div class="favicon favicon-codereview" title="Code Review Stack Exchange"></div><a href="https://codereview.stackexchange.com/questions/232935/brainfuck-interpreter-written-in-c" class="js-gps-track question-hyperlink mb0" data-gps-track="site.switch({ item_type:11, target_site:196 }); posts_hot_network.click({ item_type:2, location:11 })">
Brainfuck interpreter written in C
</a>
</li>
<li class="dno js-hidden">
<div class="favicon favicon-bitcoin" title="Bitcoin Stack Exchange"></div><a href="https://bitcoin.stackexchange.com/questions/91943/how-do-i-recover-from-a-cryptocurrency-scam" class="js-gps-track question-hyperlink mb0" data-gps-track="site.switch({ item_type:11, target_site:308 }); posts_hot_network.click({ item_type:2, location:11 })">
How do I recover from a cryptocurrency scam?
</a>
</li>
<li class="dno js-hidden">
<div class="favicon favicon-law" title="Law Stack Exchange"></div><a href="https://law.stackexchange.com/questions/46908/summary-proceeding-in-new-zealand-denying-liability-but-not-requesting-a-heari" class="js-gps-track question-hyperlink mb0" data-gps-track="site.switch({ item_type:11, target_site:617 }); posts_hot_network.click({ item_type:2, location:11 })">
Summary Proceeding in New Zealand - Denying liability but not requesting a hearing
</a>
</li>
<li class="dno js-hidden">
<div class="favicon favicon-academia" title="Academia Stack Exchange"></div><a href="https://academia.stackexchange.com/questions/140675/students-requesting-to-switch-partners-mid-term" class="js-gps-track question-hyperlink mb0" data-gps-track="site.switch({ item_type:11, target_site:415 }); posts_hot_network.click({ item_type:2, location:11 })">
Students requesting to switch partners mid term
</a>
</li>
<li class="dno js-hidden">
<div class="favicon favicon-retrocomputing" title="Retrocomputing Stack Exchange"></div><a href="https://retrocomputing.stackexchange.com/questions/12898/why-are-old-computers-so-vulnerable-to-temperature-changes-and-moisture" class="js-gps-track question-hyperlink mb0" data-gps-track="site.switch({ item_type:11, target_site:648 }); posts_hot_network.click({ item_type:2, location:11 })">
Why are old computers so vulnerable to temperature changes and moisture?
</a>
</li>
<li class="dno js-hidden">
<div class="favicon favicon-codereview" title="Code Review Stack Exchange"></div><a href="https://codereview.stackexchange.com/questions/233061/creating-an-affinity-matrix-between-protein-and-rna-sequences" class="js-gps-track question-hyperlink mb0" data-gps-track="site.switch({ item_type:11, target_site:196 }); posts_hot_network.click({ item_type:2, location:11 })">
Creating an affinity-matrix between protein and RNA sequences
</a>
</li>
<li class="dno js-hidden">
<div class="favicon favicon-gis" title="Geographic Information Systems Stack Exchange"></div><a href="https://gis.stackexchange.com/questions/343195/switching-road-names-from-uppercase-to-mixed-case-in-arcmap" class="js-gps-track question-hyperlink mb0" data-gps-track="site.switch({ item_type:11, target_site:79 }); posts_hot_network.click({ item_type:2, location:11 })">
Switching road names from uppercase to mixed case in ArcMap?
</a>
</li>
<li class="dno js-hidden">
<div class="favicon favicon-dba" title="Database Administrators Stack Exchange"></div><a href="https://dba.stackexchange.com/questions/254252/default-permissible-datatype-conversion-matrix" class="js-gps-track question-hyperlink mb0" data-gps-track="site.switch({ item_type:11, target_site:182 }); posts_hot_network.click({ item_type:2, location:11 })">
default permissible datatype conversion matrix
</a>
</li>
<li class="dno js-hidden">
<div class="favicon favicon-cooking" title="Seasoned Advice"></div><a href="https://cooking.stackexchange.com/questions/103745/how-do-you-preserve-fresh-ginger" class="js-gps-track question-hyperlink mb0" data-gps-track="site.switch({ item_type:11, target_site:49 }); posts_hot_network.click({ item_type:2, location:11 })">
How do you preserve fresh ginger?
</a>
</li>
<li class="dno js-hidden">
<div class="favicon favicon-worldbuilding" title="Worldbuilding Stack Exchange"></div><a href="https://worldbuilding.stackexchange.com/questions/162093/is-there-any-conceivable-way-to-turn-off-a-star" class="js-gps-track question-hyperlink mb0" data-gps-track="site.switch({ item_type:11, target_site:579 }); posts_hot_network.click({ item_type:2, location:11 })">
Is there any conceivable way to &quot;turn off&quot; a star?
</a>
</li>
<li class="dno js-hidden">
<div class="favicon favicon-travel" title="Travel Stack Exchange"></div><a href="https://travel.stackexchange.com/questions/150205/what-can-i-do-at-hong-kong-airport-for-13-hours" class="js-gps-track question-hyperlink mb0" data-gps-track="site.switch({ item_type:11, target_site:273 }); posts_hot_network.click({ item_type:2, location:11 })">
What can I do at Hong Kong Airport for 13 hours?
</a>
</li>
</ul>
<a href="#"
class="show-more js-show-more js-gps-track"
data-gps-track="posts_hot_network.click({ item_type:3, location:11 })">
more hot questions
</a>
</div>
<div id="feed-link" class="js-feed-link">
<a href="/feeds/question/364940" title="Feed of this question and its answers">
<svg aria-hidden="true" class="svg-icon fc-orange-400 iconRss" width="18" height="18" viewBox="0 0 18 18"><path d="M1 3c0-1.1.9-2 2-2h12a2 2 0 0 1 2 2v12a2 2 0 0 1-2 2H3a2 2 0 0 1-2-2V3zm14.5 12C15.5 8.1 9.9 2.5 3 2.5V5a10 10 0 0 1 10 10h2.5zm-5 0A7.5 7.5 0 0 0 3 7.5V10a5 5 0 0 1 5 5h2.5zm-5 0A2.5 2.5 0 0 0 3 12.5V15h2.5z"/></svg>
Question feed
</a>
</div>
<aside class="s-modal js-feed-link-modal" tabindex="-1" role="dialog" aria-labelledby="feed-modal-title" aria-describedby="feed-modal-description" aria-hidden="true">
<div class="s-modal--dialog js-modal-dialog wmx4" role="document" data-controller="se-draggable">
<h1 class="s-modal--header fw-bold js-first-tabbable" id="feed-modal-title" data-target="se-draggable.handle" tabindex="0">
Subscribe to RSS
</h1>
<div class="grid gs4 gsy fd-column">
<div class="grid--cell">
<label class="d-block s-label c-default" for="feed-url">
Question feed
<p class="s-description mt2" id="feed-modal-description">To subscribe to this RSS feed, copy and paste this URL into your RSS reader.</p>
</label>
</div>
<div class="grid ps-relative">
<input class="s-input" type="text" name="feed-url" id="feed-url" readonly="readonly" value="https://electronics.stackexchange.com/feeds/question/364940" />
<svg aria-hidden="true" class="svg-icon s-input-icon fc-orange-400 iconRss" width="18" height="18" viewBox="0 0 18 18"><path d="M1 3c0-1.1.9-2 2-2h12a2 2 0 0 1 2 2v12a2 2 0 0 1-2 2H3a2 2 0 0 1-2-2V3zm14.5 12C15.5 8.1 9.9 2.5 3 2.5V5a10 10 0 0 1 10 10h2.5zm-5 0A7.5 7.5 0 0 0 3 7.5V10a5 5 0 0 1 5 5h2.5zm-5 0A2.5 2.5 0 0 0 3 12.5V15h2.5z"/></svg>
</div>
</div>
<a class="s-modal--close s-btn s-btn__muted js-modal-close js-last-tabbable" href="#" aria-label="Close">
<svg aria-hidden="true" class="svg-icon iconClearSm" width="14" height="14" viewBox="0 0 14 14"><path d="M12 3.41L10.59 2 7 5.59 3.41 2 2 3.41 5.59 7 2 10.59 3.41 12 7 8.41 10.59 12 12 10.59 8.41 7 12 3.41z"/></svg>
</a>
</div>
</aside>
</div>
</div>
<script>StackExchange.ready(function(){$.get('/posts/364940/ivc/fc3b');});</script>
<noscript><div><img src="/posts/364940/ivc/fc3b" class="dno" alt="" width="0" height="0"></div></noscript><div style="display:none" id="prettify-lang"></div></div>
</div>
</div>
<script>;try{(function(a){function b(a){return'string'==typeof a?document.getElementById(a):a}function c(a){return a=b(a),!!a&&'none'===getComputedStyle(a).display}function d(a){return!c(a)}function e(a){return!!a}function f(a){return /^\s*$/.test(b(a).innerHTML)}function g(a){var b=a.style;b.height=b.maxHeight=b.minHeight='auto',b.display='none',[].forEach.call(a.children,g)}function h(a,b){var c;return function(){return a&&(c=a.call(b||this,arguments),a=null),c}}function i(a){var b=document.createElement('script');b.src=a,document.body.appendChild(b)}function j(a){return k([],a)}function k(a,b){return a.push=function(a){return b(),delete this.push,this.push(a)},a}function l(){try{return!new Function('return async()=>{};')}catch(a){return!0}}function m(){return'undefined'!=typeof googletag&&!!googletag.apiReady}function n(){m()||(googletag={cmd:j(A)})}function o(){var a=document.createElement('div');a.className='adsbox',a.id='clc-abd',a.style.position='absolute',a.style.pointerEvents='none',a.innerHTML='&nbsp;',document.body.appendChild(a)}function p(a){var b=a.serviceName,c=a.slot,d=a.lineItemId;try{var e=c.getSlotElementId(),f=[];e||f.push('id=0');var h=document.getElementById(e);if(!e||h?h.hasAttribute('data-clc-stalled')&&f.push('st=1'):f.push('el=0'),0!==f.length)return void B(f.join('&'));if(-1!==x.dh.indexOf(d))g(h);else if(d&&(h.setAttribute('data-clc-prefilled','true'),'dfp-msb'==e)){var i=document.getElementById('hireme');g(i)}h.setAttribute('data-clc-ready','true')}catch(a){var j=document.querySelector('#dfp-tsb, #dfp-isb, #clc-tsb');j&&j.setAttribute('data-clc-ready','true'),B('e=1')}}function q(){return Object.keys(F.ids)}function r(a){var b=F.ids[a],c=F.slots[b];'function'==typeof c&&(c=c(a));return{path:'/'+C+'/'+E+'/'+b+'/'+D,sizes:c,zone:b}}function s(a){return!(clc.collapse&&void 0!==clc.collapse[a])||!!clc.collapse[a]}function t(a,b){'dfp-isb'===a&&b.setTargeting('Sidebar',['Inline']),'dfp-tsb'===a&&b.setTargeting('Sidebar',['Right']);var c=r(a),d=c.path,e=c.sizes,f=c.zone,g=googletag.defineSlot(d,e,a);s(f)&&g.setCollapseEmptyDiv(!0,!0),g.addService(b),!1}function u(b){var c=a.dfp&&a.dfp.targeting||{};Object.keys(c).forEach(function(a){b.setTargeting(a,c[a])})}function v(a){var g=a.map(b).filter(e);return{eligible:g.filter(f).filter(d),ineligible:g.filter(c)}}function w(b){if(void 0===b&&(b=q()),!m())return n(),void googletag.cmd.push(function(){return w(b)});var c=v(b),d=c.eligible,e=c.ineligible;if(e.forEach(g),0!==d.length){x.abd&&o(),googletag.destroySlots();var f=googletag.pubads();x.sf&&(f.setForceSafeFrame(!0),f.setSafeFrameConfig({allowOverlayExpansion:!0,allowPushExpansion:!0,sandbox:!0})),f.enableSingleRequest(),a.sreEvent||(f.addEventListener('slotRenderEnded',p),a.sreEvent=!0),u(f),d.forEach(function(a){t(a.id,f),a.setAttribute('data-dfp-zone','true')}),googletag.enableServices(),d.forEach(function(a){googletag.display(a.id)})}}var x=function(a){for(var b=[],c=1;c<arguments.length;c++)b[c-1]=arguments[c];for(var d,e=0,f=b;e<f.length;e++)for(var g in d=f[e],d)a[g]=d[g];return a}({"lib":"https://cdn.sstatic.net/clc/clc.min.js?v=b34823b15e14","style":"https://cdn.sstatic.net/clc/styles/clc.min.css?v=aab7c1365319","u":"https://clc.stackoverflow.com/markup.js","wa":true,"kt":2000,"tto":true,"h":"clc.stackoverflow.com","allowed":"^(((talent\\.)?stackoverflow)|(blog\\.codinghorror)|(serverfault|askubuntu)|([^\\.]+\\.stackexchange))\\.com$","wv":true,"al":false,"dh":[5171832659],"abd":true},a.options||{}),y=h(function(){var a=x.lib;l()&&(a=a.replace(/(\.min)?\.js(\?v=[0-9a-fA-F]+)?$/,'.ie$1.js$2')),i(a)}),z=a.cmd||[];Array.isArray(z)&&(0<z.length?y():k(z,y));var A=h(function(){i('https://www.googletagservices.com/tag/js/gpt.js')}),B=function(a){new Image().src='https://'+x.h+'/stalled.gif?'+a},C='248424177',D=/^\/tags\//.test(location.pathname)||/^\/questions\/tagged\//.test(location.pathname)?'tag-pages':/^\/$/.test(location.pathname)||/^\/home/.test(location.pathname)?'home-page':'question-pages',E=location.hostname;var F={slots:{lb:[[728,90]],mlb:[[728,90]],smlb:[[728,90]],bmlb:[[728,90]],sb:function(a){return'dfp-tsb'===a?[[300,250],[300,600]]:[[300,250]]},"tag-sponsorship":[[730,135]],"mobile-below-question":[[320,50],[300,250]],msb:[[300,250],[300,600]]},ids:{"dfp-tlb":'lb',"dfp-mlb":'mlb',"dfp-smlb":'smlb',"dfp-bmlb":'bmlb',"dfp-tsb":'sb',"dfp-isb":'sb',"dfp-tag":'tag-sponsorship',"dfp-msb":'msb',"dfp-m-aq":'mobile-below-question',"clc-tlb":'lb',"clc-mlb":'mlb',"clc-tsb":'sb'}};(function(){var b=x.al;b&&z.push(function(){return a.load()})})(),n(),a.dfp={load:w},a.options=x,a.cmd=z})(this.clc=this.clc||{})}catch(a){window.console.error(a)}</script> <script>
var clc = clc || {};
clc.collapse = { sb: !0, 'tag-sponsorship': !0, lb:!0, mlb:!0, smlb:!0, bmlb:!0, 'mobile-below-question':!0};
clc.options = clc.options || {};
clc.options.sf = !1;
clc.options.hb = !1;
clc.dfp = clc.dfp || {};
clc.dfp.targeting = {Registered:['false'],'ron-tag':['voltage-reference']};
var googletag = googletag || {};
googletag.cmd = googletag.cmd || [];
googletag.cmd.push(function () { clc.dfp.load(); });
</script>
<footer id="footer" class="site-footer js-footer" role="contentinfo">
<div class="site-footer--container">
<nav class="site-footer--nav">
<div class="site-footer--col site-footer--col__visible js-footer-col" data-name="default">
<h5 class="-title"><a href="/">Electrical Engineering</a></h5>
<ul class="-list js-primary-footer-links">
<li class="-item"><a class="js-gps-track -link" data-gps-track="footer.click({ location: 2, link: 2 })" href="/tour">Tour</a></li>
<li class="-item"><a href="/help" class="js-gps-track -link" data-gps-track="footer.click({ location: 2, link: 3 })">Help</a></li>
<li class="-item"><a class="js-gps-track -link" data-gps-track="footer.click({ location: 2, link: 5 })" href="https://chat.stackexchange.com?tab=site&host=electronics.stackexchange.com">Chat</a></li>
<li class="-item"><a class="js-gps-track -link" data-gps-track="footer.click({ location: 2, link: 13 })" href="/contact">Contact</a></li>
<li class="-item"><a class="js-gps-track -link" data-gps-track="footer.click({ location: 2, link: 14 })" href="https://electronics.meta.stackexchange.com">Feedback</a></li>
<li class="-item"><a onclick='StackExchange.switchMobile("on")' class="js-gps-track -link" data-gps-track="footer.click({ location: 2, link: 12 })">Mobile</a></li>
</ul>
</div>
<div class="site-footer--col site-footer--col__visible js-footer-col" data-name="default">
<h5 class="-title"><a class="js-gps-track" data-gps-track="footer.click({ location: 2, link: 1 })" href="https://stackoverflow.com/company/about">Company</a></h5>
<ul class="-list">
<li class="-item"><a href="https://stackoverflow.com" class="js-gps-track -link" data-gps-track="footer.click({ location: 2, link: 15})">Stack Overflow</a></li>
<li class="-item"><a href="https://www.stackoverflowbusiness.com" class="js-gps-track -link" data-gps-track="footer.click({ location: 2, link: 19 })">Stack Overflow Business</a></li>
<li class="-item"><a href="https://stackoverflow.com/jobs" class="js-gps-track -link" data-gps-track="footer.click({ location: 2, link: 17})">Developer Jobs</a></li>
<li class="-item"><a class="js-gps-track -link" data-gps-track="footer.click({ location: 2, link: 1 })" href="https://stackoverflow.com/company/about">About</a></li>
<li class="-item"><a class="js-gps-track -link" data-gps-track="footer.click({ location: 2, link: 27 })" href="https://stackoverflow.com/company/press">Press</a></li>
<li class="-item"><a class="js-gps-track -link" data-gps-track="footer.click({ location: 2, link: 7 })" href="https://stackoverflow.com/legal">Legal</a></li>
<li class="-item"><a class="js-gps-track -link" data-gps-track="footer.click({ location: 2, link: 8 })" href="https://stackoverflow.com/legal/privacy-policy">Privacy Policy</a></li>
</ul>
</div>
<div class="site-footer--col site-footer--categories-nav site-footer--col__visible">
<a href="#" class="site-footer--back js-footer-back"><svg aria-hidden="true" class="svg-icon iconArrowLeftAlt" width="18" height="18" viewBox="0 0 18 18"><path d="M10.58 16L12 14.59 6.4 9 12 3.41 10.57 2l-7 7 7 7z"/></svg></a>
<div>
<h5 class="-title"><a href="https://stackexchange.com" data-gps-track="footer.click({ location: 2, link: 30 })">Stack Exchange<br> Network</a></h5>
<ul class="-list">
<li class="-item"><a href="#" class="-link _expandable js-footer-category-trigger js-gps-track" data-gps-track="footer.click({ location: 2, link: 24 })" data-target="Technology">Technology</a></li>
<li class="-item"><a href="#" class="-link _expandable js-footer-category-trigger js-gps-track" data-gps-track="footer.click({ location: 2, link: 24 })" data-target="Life / Arts">Life / Arts</a></li>
<li class="-item"><a href="#" class="-link _expandable js-footer-category-trigger js-gps-track" data-gps-track="footer.click({ location: 2, link: 24 })" data-target="Culture / Recreation">Culture / Recreation</a></li>
<li class="-item"><a href="#" class="-link _expandable js-footer-category-trigger js-gps-track" data-gps-track="footer.click({ location: 2, link: 24 })" data-target="Science">Science</a></li>
<li class="-item"><a href="#" class="-link _expandable js-footer-category-trigger js-gps-track" data-gps-track="footer.click({ location: 2, link: 24 })" data-target="Other">Other</a></li>
</ul>
</div>
</div>
<div class="site-footer--categories">
<div class="site-footer--col site-footer--category js-footer-col" data-name="Technology">
<ul class="-list">
<li class="-item"><a href="https://stackoverflow.com" class="-link js-gps-track" data-gps-track="footer.click({ location: 2, link: 25 })" title="professional and enthusiast programmers">Stack Overflow</a></li>
<li class="-item"><a href="https://serverfault.com" class="-link js-gps-track" data-gps-track="footer.click({ location: 2, link: 25 })" title="system and network administrators">Server Fault</a></li>
<li class="-item"><a href="https://superuser.com" class="-link js-gps-track" data-gps-track="footer.click({ location: 2, link: 25 })" title="computer enthusiasts and power users">Super User</a></li>
<li class="-item"><a href="https://webapps.stackexchange.com" class="-link js-gps-track" data-gps-track="footer.click({ location: 2, link: 25 })" title="power users of web applications">Web Applications</a></li>
<li class="-item"><a href="https://askubuntu.com" class="-link js-gps-track" data-gps-track="footer.click({ location: 2, link: 25 })" title="Ubuntu users and developers">Ask Ubuntu</a></li>
<li class="-item"><a href="https://webmasters.stackexchange.com" class="-link js-gps-track" data-gps-track="footer.click({ location: 2, link: 25 })" title="pro webmasters">Webmasters</a></li>
<li class="-item"><a href="https://gamedev.stackexchange.com" class="-link js-gps-track" data-gps-track="footer.click({ location: 2, link: 25 })" title="professional and independent game developers">Game Development</a></li>
</ul></div><div class="site-footer--col site-footer--category js-footer-col" data-name="Technology"><ul class="-list">
<li class="-item"><a href="https://tex.stackexchange.com" class="-link js-gps-track" data-gps-track="footer.click({ location: 2, link: 25 })" title="users of TeX, LaTeX, ConTeXt, and related typesetting systems">TeX - LaTeX</a></li>
<li class="-item"><a href="https://softwareengineering.stackexchange.com" class="-link js-gps-track" data-gps-track="footer.click({ location: 2, link: 25 })" title="professionals, academics, and students working within the systems development life cycle">Software Engineering</a></li>
<li class="-item"><a href="https://unix.stackexchange.com" class="-link js-gps-track" data-gps-track="footer.click({ location: 2, link: 25 })" title="users of Linux, FreeBSD and other Un*x-like operating systems">Unix &amp; Linux</a></li>
<li class="-item"><a href="https://apple.stackexchange.com" class="-link js-gps-track" data-gps-track="footer.click({ location: 2, link: 25 })" title="power users of Apple hardware and software">Ask Different (Apple)</a></li>
<li class="-item"><a href="https://wordpress.stackexchange.com" class="-link js-gps-track" data-gps-track="footer.click({ location: 2, link: 25 })" title="WordPress developers and administrators">WordPress Development</a></li>
<li class="-item"><a href="https://gis.stackexchange.com" class="-link js-gps-track" data-gps-track="footer.click({ location: 2, link: 25 })" title="cartographers, geographers and GIS professionals">Geographic Information Systems</a></li>
<li class="-item"><a href="https://electronics.stackexchange.com" class="-link js-gps-track" data-gps-track="footer.click({ location: 2, link: 25 })" title="electronics and electrical engineering professionals, students, and enthusiasts">Electrical Engineering</a></li>
</ul></div><div class="site-footer--col site-footer--category js-footer-col" data-name="Technology"><ul class="-list">
<li class="-item"><a href="https://android.stackexchange.com" class="-link js-gps-track" data-gps-track="footer.click({ location: 2, link: 25 })" title="enthusiasts and power users of the Android operating system">Android Enthusiasts</a></li>
<li class="-item"><a href="https://security.stackexchange.com" class="-link js-gps-track" data-gps-track="footer.click({ location: 2, link: 25 })" title="information security professionals">Information Security</a></li>
<li class="-item"><a href="https://dba.stackexchange.com" class="-link js-gps-track" data-gps-track="footer.click({ location: 2, link: 25 })" title="database professionals who wish to improve their database skills and learn from others in the community">Database Administrators</a></li>
<li class="-item"><a href="https://drupal.stackexchange.com" class="-link js-gps-track" data-gps-track="footer.click({ location: 2, link: 25 })" title="Drupal developers and administrators">Drupal Answers</a></li>
<li class="-item"><a href="https://sharepoint.stackexchange.com" class="-link js-gps-track" data-gps-track="footer.click({ location: 2, link: 25 })" title="SharePoint enthusiasts">SharePoint</a></li>
<li class="-item"><a href="https://ux.stackexchange.com" class="-link js-gps-track" data-gps-track="footer.click({ location: 2, link: 25 })" title="user experience researchers and experts">User Experience</a></li>
<li class="-item"><a href="https://mathematica.stackexchange.com" class="-link js-gps-track" data-gps-track="footer.click({ location: 2, link: 25 })" title="users of Wolfram Mathematica">Mathematica</a></li>
</ul></div><div class="site-footer--col site-footer--category js-footer-col" data-name="Technology"><ul class="-list">
<li class="-item"><a href="https://salesforce.stackexchange.com" class="-link js-gps-track" data-gps-track="footer.click({ location: 2, link: 25 })" title="Salesforce administrators, implementation experts, developers and anybody in-between">Salesforce</a></li>
<li class="-item"><a href="https://expressionengine.stackexchange.com" class="-link js-gps-track" data-gps-track="footer.click({ location: 2, link: 25 })" title="administrators, end users, developers and designers for ExpressionEngine&#174; CMS">ExpressionEngine&#174; Answers</a></li>
<li class="-item"><a href="https://pt.stackoverflow.com" class="-link js-gps-track" data-gps-track="footer.click({ location: 2, link: 25 })" title="programadores profissionais e entusiastas">Stack Overflow em Portugu&#234;s</a></li>
<li class="-item"><a href="https://blender.stackexchange.com" class="-link js-gps-track" data-gps-track="footer.click({ location: 2, link: 25 })" title="people who use Blender to create 3D graphics, animations, or games">Blender</a></li>
<li class="-item"><a href="https://networkengineering.stackexchange.com" class="-link js-gps-track" data-gps-track="footer.click({ location: 2, link: 25 })" title="network engineers">Network Engineering</a></li>
<li class="-item"><a href="https://crypto.stackexchange.com" class="-link js-gps-track" data-gps-track="footer.click({ location: 2, link: 25 })" title="software developers, mathematicians and others interested in cryptography">Cryptography</a></li>
<li class="-item"><a href="https://codereview.stackexchange.com" class="-link js-gps-track" data-gps-track="footer.click({ location: 2, link: 25 })" title="peer programmer code reviews">Code Review</a></li>
</ul></div><div class="site-footer--col site-footer--category js-footer-col" data-name="Technology"><ul class="-list">
<li class="-item"><a href="https://magento.stackexchange.com" class="-link js-gps-track" data-gps-track="footer.click({ location: 2, link: 25 })" title="users of the Magento e-Commerce platform">Magento</a></li>
<li class="-item"><a href="https://softwarerecs.stackexchange.com" class="-link js-gps-track" data-gps-track="footer.click({ location: 2, link: 25 })" title="people seeking specific software recommendations">Software Recommendations</a></li>
<li class="-item"><a href="https://dsp.stackexchange.com" class="-link js-gps-track" data-gps-track="footer.click({ location: 2, link: 25 })" title="practitioners of the art and science of signal, image and video processing">Signal Processing</a></li>
<li class="-item"><a href="https://emacs.stackexchange.com" class="-link js-gps-track" data-gps-track="footer.click({ location: 2, link: 25 })" title="those using, extending or developing Emacs">Emacs</a></li>
<li class="-item"><a href="https://raspberrypi.stackexchange.com" class="-link js-gps-track" data-gps-track="footer.click({ location: 2, link: 25 })" title="users and developers of hardware and software for Raspberry Pi">Raspberry Pi</a></li>
<li class="-item"><a href="https://ru.stackoverflow.com" class="-link js-gps-track" data-gps-track="footer.click({ location: 2, link: 25 })" title="программистов">Stack Overflow на русском</a></li>
<li class="-item"><a href="https://codegolf.stackexchange.com" class="-link js-gps-track" data-gps-track="footer.click({ location: 2, link: 25 })" title="programming puzzle enthusiasts and code golfers">Code Golf</a></li>
</ul></div><div class="site-footer--col site-footer--category js-footer-col" data-name="Technology"><ul class="-list">
<li class="-item"><a href="https://es.stackoverflow.com" class="-link js-gps-track" data-gps-track="footer.click({ location: 2, link: 25 })" title="programadores y profesionales de la inform&#225;tica">Stack Overflow en espa&#241;ol</a></li>
<li class="-item"><a href="https://ethereum.stackexchange.com" class="-link js-gps-track" data-gps-track="footer.click({ location: 2, link: 25 })" title="users of Ethereum, the decentralized application platform and smart contract enabled blockchain">Ethereum</a></li>
<li class="-item"><a href="https://datascience.stackexchange.com" class="-link js-gps-track" data-gps-track="footer.click({ location: 2, link: 25 })" title="Data science professionals, Machine Learning specialists, and those interested in learning more about the field">Data Science</a></li>
<li class="-item"><a href="https://arduino.stackexchange.com" class="-link js-gps-track" data-gps-track="footer.click({ location: 2, link: 25 })" title="developers of open-source hardware and software that is compatible with Arduino">Arduino</a></li>
<li class="-item"><a href="https://bitcoin.stackexchange.com" class="-link js-gps-track" data-gps-track="footer.click({ location: 2, link: 25 })" title="Bitcoin crypto-currency enthusiasts">Bitcoin</a></li>
<li class="-item"><a href="https://sqa.stackexchange.com" class="-link js-gps-track" data-gps-track="footer.click({ location: 2, link: 25 })" title="software quality control experts, automation engineers, and software testers">Software Quality Assurance &amp; Testing</a></li>
<li class="-item"><a href="https://sound.stackexchange.com" class="-link js-gps-track" data-gps-track="footer.click({ location: 2, link: 25 })" title="sound engineers, producers, editors, and enthusiasts">Sound Design</a></li>
</ul></div><div class="site-footer--col site-footer--category js-footer-col" data-name="Technology"><ul class="-list">
<li class="-item"><a href="https://windowsphone.stackexchange.com" class="-link js-gps-track" data-gps-track="footer.click({ location: 2, link: 25 })" title="enthusiasts and power users of Windows Phone OS">Windows Phone</a></li>
<li class="-item">
<a href="https://stackexchange.com/sites#technology" class="-link js-gps-track" data-gps-track="footer.click({ location: 2, link: 26 })">
<strong>
more (27)
</strong>
</a>
</li>
</ul>
</div>
<div class="site-footer--col site-footer--category js-footer-col" data-name="Life / Arts">
<ul class="-list">
<li class="-item"><a href="https://photo.stackexchange.com" class="-link js-gps-track" data-gps-track="footer.click({ location: 2, link: 25 })" title="professional, enthusiast and amateur photographers">Photography</a></li>
<li class="-item"><a href="https://scifi.stackexchange.com" class="-link js-gps-track" data-gps-track="footer.click({ location: 2, link: 25 })" title="science fiction and fantasy enthusiasts">Science Fiction &amp; Fantasy</a></li>
<li class="-item"><a href="https://graphicdesign.stackexchange.com" class="-link js-gps-track" data-gps-track="footer.click({ location: 2, link: 25 })" title="Graphic Design professionals, students, and enthusiasts">Graphic Design</a></li>
<li class="-item"><a href="https://movies.stackexchange.com" class="-link js-gps-track" data-gps-track="footer.click({ location: 2, link: 25 })" title="movie and tv enthusiasts">Movies &amp; TV</a></li>
<li class="-item"><a href="https://music.stackexchange.com" class="-link js-gps-track" data-gps-track="footer.click({ location: 2, link: 25 })" title="musicians, students, and enthusiasts">Music: Practice &amp; Theory</a></li>
<li class="-item"><a href="https://worldbuilding.stackexchange.com" class="-link js-gps-track" data-gps-track="footer.click({ location: 2, link: 25 })" title="writers/artists using science, geography and culture to construct imaginary worlds and settings">Worldbuilding</a></li>
<li class="-item"><a href="https://video.stackexchange.com" class="-link js-gps-track" data-gps-track="footer.click({ location: 2, link: 25 })" title="engineers, producers, editors, and enthusiasts spanning the fields of video, and media creation">Video Production</a></li>
</ul></div><div class="site-footer--col site-footer--category js-footer-col" data-name="Life / Arts"><ul class="-list">
<li class="-item"><a href="https://cooking.stackexchange.com" class="-link js-gps-track" data-gps-track="footer.click({ location: 2, link: 25 })" title="professional and amateur chefs">Seasoned Advice (cooking)</a></li>
<li class="-item"><a href="https://diy.stackexchange.com" class="-link js-gps-track" data-gps-track="footer.click({ location: 2, link: 25 })" title="contractors and serious DIYers">Home Improvement</a></li>
<li class="-item"><a href="https://money.stackexchange.com" class="-link js-gps-track" data-gps-track="footer.click({ location: 2, link: 25 })" title="people who want to be financially literate">Personal Finance &amp; Money</a></li>
<li class="-item"><a href="https://academia.stackexchange.com" class="-link js-gps-track" data-gps-track="footer.click({ location: 2, link: 25 })" title="academics and those enrolled in higher education">Academia</a></li>
<li class="-item"><a href="https://law.stackexchange.com" class="-link js-gps-track" data-gps-track="footer.click({ location: 2, link: 25 })" title="legal professionals, students, and others with experience or interest in law">Law</a></li>
<li class="-item"><a href="https://fitness.stackexchange.com" class="-link js-gps-track" data-gps-track="footer.click({ location: 2, link: 25 })" title="physical fitness professionals, athletes, trainers, and those providing health-related needs">Physical Fitness</a></li>
<li class="-item"><a href="https://gardening.stackexchange.com" class="-link js-gps-track" data-gps-track="footer.click({ location: 2, link: 25 })" title="gardeners and landscapers">Gardening &amp; Landscaping</a></li>
</ul></div><div class="site-footer--col site-footer--category js-footer-col" data-name="Life / Arts"><ul class="-list">
<li class="-item"><a href="https://parenting.stackexchange.com" class="-link js-gps-track" data-gps-track="footer.click({ location: 2, link: 25 })" title="parents, grandparents, nannies and others with a parenting role">Parenting</a></li>
<li class="-item">
<a href="https://stackexchange.com/sites#lifearts" class="-link js-gps-track" data-gps-track="footer.click({ location: 2, link: 26 })">
<strong>
more (11)
</strong>
</a>
</li>
</ul>
</div>
<div class="site-footer--col site-footer--category js-footer-col" data-name="Culture / Recreation">
<ul class="-list">
<li class="-item"><a href="https://english.stackexchange.com" class="-link js-gps-track" data-gps-track="footer.click({ location: 2, link: 25 })" title="linguists, etymologists, and serious English language enthusiasts">English Language &amp; Usage</a></li>
<li class="-item"><a href="https://skeptics.stackexchange.com" class="-link js-gps-track" data-gps-track="footer.click({ location: 2, link: 25 })" title="scientific skepticism">Skeptics</a></li>
<li class="-item"><a href="https://judaism.stackexchange.com" class="-link js-gps-track" data-gps-track="footer.click({ location: 2, link: 25 })" title="those who base their lives on Jewish law and tradition and anyone interested in learning more">Mi Yodeya (Judaism)</a></li>
<li class="-item"><a href="https://travel.stackexchange.com" class="-link js-gps-track" data-gps-track="footer.click({ location: 2, link: 25 })" title="road warriors and seasoned travelers">Travel</a></li>
<li class="-item"><a href="https://christianity.stackexchange.com" class="-link js-gps-track" data-gps-track="footer.click({ location: 2, link: 25 })" title="committed Christians, experts in Christianity and those interested in learning more">Christianity</a></li>
<li class="-item"><a href="https://ell.stackexchange.com" class="-link js-gps-track" data-gps-track="footer.click({ location: 2, link: 25 })" title="speakers of other languages learning English">English Language Learners</a></li>
<li class="-item"><a href="https://japanese.stackexchange.com" class="-link js-gps-track" data-gps-track="footer.click({ location: 2, link: 25 })" title="students, teachers, and linguists wanting to discuss the finer points of the Japanese language">Japanese Language</a></li>
</ul></div><div class="site-footer--col site-footer--category js-footer-col" data-name="Culture / Recreation"><ul class="-list">
<li class="-item"><a href="https://chinese.stackexchange.com" class="-link js-gps-track" data-gps-track="footer.click({ location: 2, link: 25 })" title="students, teachers, and linguists wanting to discuss the finer points of the Chinese language">Chinese Language</a></li>
<li class="-item"><a href="https://french.stackexchange.com" class="-link js-gps-track" data-gps-track="footer.click({ location: 2, link: 25 })" title="students, teachers, and linguists wanting to discuss the finer points of the French language">French Language</a></li>
<li class="-item"><a href="https://german.stackexchange.com" class="-link js-gps-track" data-gps-track="footer.click({ location: 2, link: 25 })" title="speakers of German wanting to discuss the finer points of the language and translation">German Language</a></li>
<li class="-item"><a href="https://hermeneutics.stackexchange.com" class="-link js-gps-track" data-gps-track="footer.click({ location: 2, link: 25 })" title="professors, theologians, and those interested in exegetical analysis of biblical texts">Biblical Hermeneutics</a></li>
<li class="-item"><a href="https://history.stackexchange.com" class="-link js-gps-track" data-gps-track="footer.click({ location: 2, link: 25 })" title="historians and history buffs">History</a></li>
<li class="-item"><a href="https://spanish.stackexchange.com" class="-link js-gps-track" data-gps-track="footer.click({ location: 2, link: 25 })" title="linguists, teachers, students and Spanish language enthusiasts in general wanting to discuss the finer points of the language">Spanish Language</a></li>
<li class="-item"><a href="https://islam.stackexchange.com" class="-link js-gps-track" data-gps-track="footer.click({ location: 2, link: 25 })" title="Muslims, experts in Islam, and those interested in learning more about Islam">Islam</a></li>
</ul></div><div class="site-footer--col site-footer--category js-footer-col" data-name="Culture / Recreation"><ul class="-list">
<li class="-item"><a href="https://rus.stackexchange.com" class="-link js-gps-track" data-gps-track="footer.click({ location: 2, link: 25 })" title="лингвистов и энтузиастов русского языка">Русский язык</a></li>
<li class="-item"><a href="https://russian.stackexchange.com" class="-link js-gps-track" data-gps-track="footer.click({ location: 2, link: 25 })" title="students, teachers, and linguists wanting to discuss the finer points of the Russian language">Russian Language</a></li>
<li class="-item"><a href="https://gaming.stackexchange.com" class="-link js-gps-track" data-gps-track="footer.click({ location: 2, link: 25 })" title="passionate videogamers on all platforms">Arqade (gaming)</a></li>
<li class="-item"><a href="https://bicycles.stackexchange.com" class="-link js-gps-track" data-gps-track="footer.click({ location: 2, link: 25 })" title="people who build and repair bicycles, people who train cycling, or commute on bicycles">Bicycles</a></li>
<li class="-item"><a href="https://rpg.stackexchange.com" class="-link js-gps-track" data-gps-track="footer.click({ location: 2, link: 25 })" title="gamemasters and players of tabletop, paper-and-pencil role-playing games">Role-playing Games</a></li>
<li class="-item"><a href="https://anime.stackexchange.com" class="-link js-gps-track" data-gps-track="footer.click({ location: 2, link: 25 })" title="anime and manga fans">Anime &amp; Manga</a></li>
<li class="-item"><a href="https://puzzling.stackexchange.com" class="-link js-gps-track" data-gps-track="footer.click({ location: 2, link: 25 })" title="those who create, solve, and study puzzles">Puzzling</a></li>
</ul></div><div class="site-footer--col site-footer--category js-footer-col" data-name="Culture / Recreation"><ul class="-list">
<li class="-item"><a href="https://mechanics.stackexchange.com" class="-link js-gps-track" data-gps-track="footer.click({ location: 2, link: 25 })" title="mechanics and DIY enthusiast owners of cars, trucks, and motorcycles">Motor Vehicle Maintenance &amp; Repair</a></li>
<li class="-item"><a href="https://boardgames.stackexchange.com" class="-link js-gps-track" data-gps-track="footer.click({ location: 2, link: 25 })" title="people who like playing board games, designing board games or modifying the rules of existing board games">Board &amp; Card Games</a></li>
<li class="-item"><a href="https://bricks.stackexchange.com" class="-link js-gps-track" data-gps-track="footer.click({ location: 2, link: 25 })" title="LEGO&#174; and building block enthusiasts">Bricks</a></li>
<li class="-item"><a href="https://homebrew.stackexchange.com" class="-link js-gps-track" data-gps-track="footer.click({ location: 2, link: 25 })" title="dedicated home brewers and serious enthusiasts">Homebrewing</a></li>
<li class="-item"><a href="https://martialarts.stackexchange.com" class="-link js-gps-track" data-gps-track="footer.click({ location: 2, link: 25 })" title="students and teachers of all martial arts">Martial Arts</a></li>
<li class="-item"><a href="https://outdoors.stackexchange.com" class="-link js-gps-track" data-gps-track="footer.click({ location: 2, link: 25 })" title="people who love being outdoors enjoying nature and wilderness, and learning about the required skills and equipment">The Great Outdoors</a></li>
<li class="-item"><a href="https://poker.stackexchange.com" class="-link js-gps-track" data-gps-track="footer.click({ location: 2, link: 25 })" title="serious players and enthusiasts of poker">Poker</a></li>
</ul></div><div class="site-footer--col site-footer--category js-footer-col" data-name="Culture / Recreation"><ul class="-list">
<li class="-item"><a href="https://chess.stackexchange.com" class="-link js-gps-track" data-gps-track="footer.click({ location: 2, link: 25 })" title="serious players and enthusiasts of chess">Chess</a></li>
<li class="-item"><a href="https://sports.stackexchange.com" class="-link js-gps-track" data-gps-track="footer.click({ location: 2, link: 25 })" title="participants in team and individual sport activities">Sports</a></li>
<li class="-item">
<a href="https://stackexchange.com/sites#culturerecreation" class="-link js-gps-track" data-gps-track="footer.click({ location: 2, link: 26 })">
<strong>
more (16)
</strong>
</a>
</li>
</ul>
</div>
<div class="site-footer--col site-footer--category js-footer-col" data-name="Science">
<ul class="-list">
<li class="-item"><a href="https://mathoverflow.net" class="-link js-gps-track" data-gps-track="footer.click({ location: 2, link: 25 })" title="professional mathematicians">MathOverflow</a></li>
<li class="-item"><a href="https://math.stackexchange.com" class="-link js-gps-track" data-gps-track="footer.click({ location: 2, link: 25 })" title="people studying math at any level and professionals in related fields">Mathematics</a></li>
<li class="-item"><a href="https://stats.stackexchange.com" class="-link js-gps-track" data-gps-track="footer.click({ location: 2, link: 25 })" title="people interested in statistics, machine learning, data analysis, data mining, and data visualization">Cross Validated (stats)</a></li>
<li class="-item"><a href="https://cstheory.stackexchange.com" class="-link js-gps-track" data-gps-track="footer.click({ location: 2, link: 25 })" title="theoretical computer scientists and researchers in related fields">Theoretical Computer Science</a></li>
<li class="-item"><a href="https://physics.stackexchange.com" class="-link js-gps-track" data-gps-track="footer.click({ location: 2, link: 25 })" title="active researchers, academics and students of physics">Physics</a></li>
<li class="-item"><a href="https://chemistry.stackexchange.com" class="-link js-gps-track" data-gps-track="footer.click({ location: 2, link: 25 })" title="scientists, academics, teachers, and students in the field of chemistry">Chemistry</a></li>
<li class="-item"><a href="https://biology.stackexchange.com" class="-link js-gps-track" data-gps-track="footer.click({ location: 2, link: 25 })" title="biology researchers, academics, and students">Biology</a></li>
</ul></div><div class="site-footer--col site-footer--category js-footer-col" data-name="Science"><ul class="-list">
<li class="-item"><a href="https://cs.stackexchange.com" class="-link js-gps-track" data-gps-track="footer.click({ location: 2, link: 25 })" title="students, researchers and practitioners of computer science">Computer Science</a></li>
<li class="-item"><a href="https://philosophy.stackexchange.com" class="-link js-gps-track" data-gps-track="footer.click({ location: 2, link: 25 })" title="those interested in the study of the fundamental nature of knowledge, reality, and existence">Philosophy</a></li>
<li class="-item"><a href="https://linguistics.stackexchange.com" class="-link js-gps-track" data-gps-track="footer.click({ location: 2, link: 25 })" title="professional linguists and others with an interest in linguistic research and theory">Linguistics</a></li>
<li class="-item"><a href="https://psychology.stackexchange.com" class="-link js-gps-track" data-gps-track="footer.click({ location: 2, link: 25 })" title="practitioners, researchers, and students in cognitive science, psychology, neuroscience, and psychiatry">Psychology &amp; Neuroscience</a></li>
<li class="-item"><a href="https://scicomp.stackexchange.com" class="-link js-gps-track" data-gps-track="footer.click({ location: 2, link: 25 })" title="scientists using computers to solve scientific problems">Computational Science</a></li>
<li class="-item">
<a href="https://stackexchange.com/sites#science" class="-link js-gps-track" data-gps-track="footer.click({ location: 2, link: 26 })">
<strong>
more (8)
</strong>
</a>
</li>
</ul>
</div>
<div class="site-footer--col site-footer--category js-footer-col" data-name="Other">
<ul class="-list">
<li class="-item"><a href="https://meta.stackexchange.com" class="-link js-gps-track" data-gps-track="footer.click({ location: 2, link: 25 })" title="meta-discussion of the Stack Exchange family of Q&amp;A websites">Meta Stack Exchange</a></li>
<li class="-item"><a href="https://stackapps.com" class="-link js-gps-track" data-gps-track="footer.click({ location: 2, link: 25 })" title="apps, scripts, and development with the Stack Exchange API">Stack Apps</a></li>
<li class="-item"><a href="https://api.stackexchange.com" class="-link js-gps-track" data-gps-track="footer.click({ location: 2, link: 25 })" title="programmatic interaction with Stack Exchange sites">API</a></li>
<li class="-item"><a href="https://data.stackexchange.com" class="-link js-gps-track" data-gps-track="footer.click({ location: 2, link: 25 })" title="querying Stack Exchange data using SQL">Data</a></li>
</ul>
</div>
</div>
</nav>
<div class="site-footer--copyright fs-fine">
<ul class="-list">
<li class="-item"><a class="js-gps-track -link" data-gps-track="footer.click({ location: 2, link:4 })" href="https://stackoverflow.blog?blb=1">Blog</a></li>
<li class="-item"><a href="https://www.facebook.com/officialstackoverflow/" class="-link js-gps-track" data-gps-track="footer.click({ location: 2, link: 31 })">Facebook</a></li>
<li class="-item"><a href="https://twitter.com/stackoverflow" class="-link js-gps-track" data-gps-track="footer.click({ location: 2, link: 32 })">Twitter</a></li>
<li class="-item"><a href="https://linkedin.com/company/stack-overflow" class="-link js-gps-track" data-gps-track="footer.click({ location: 2, link: 33 })">LinkedIn</a></li>
</ul>
<p class="mt-auto mb24">
site design / logo &#169; 2019 Stack Exchange Inc; user contributions licensed under <a href="https://creativecommons.org/licenses/by-sa/4.0/" class="js-gps-track" data-gps-track="footer.click({ location: 2, link: 34 })" rel="license">cc by-sa 4.0</a>
with <a href="https://stackoverflow.blog/2009/06/25/attribution-required/" class="js-gps-track" data-gps-track="footer.click({ location: 2, link:35 })" rel="license">attribution required</a>.
<span id="svnrev">rev&nbsp;2019.11.27.35508</span>
</p>
</div>
</div>
</footer>
<script>StackExchange.ready(function () { StackExchange.responsiveness.addSwitcher(); })</script>
<noscript>
<div id="noscript-warning">Electrical Engineering Stack Exchange works best with JavaScript enabled
<img src="https://pixel.quantserve.com/pixel/p-c1rF4kxgLUzNc.gif" alt="" class="dno">
</div>
</noscript>
<script>
(function(i, s, o, g, r, a, m) {
i['GoogleAnalyticsObject'] = r; i[r] = i[r] || function() { (i[r].q = i[r].q || []).push(arguments) }, i[r].l = 1 * new Date(); a = s.createElement(o),
m = s.getElementsByTagName(o)[0]; a.async = 1; a.src = g; m.parentNode.insertBefore(a, m);
})(window, document, 'script', 'https://www.google-analytics.com/analytics.js', 'ga');
StackExchange.ready(function () {
StackExchange.ga.init({
sendTitles: true,
tracker: window.ga,
trackingCodes: [
'UA-108242619-5'
],
checkDimension: 'dimension42'
});
StackExchange.ga.setDimension('dimension2', '|voltage-reference|');
StackExchange.ga.setDimension('dimension3', 'Questions/Show');
StackExchange.ga.trackPageView();
});
/**/
var _qevents = _qevents || [],
_comscore = _comscore || [];
(function() {
var ssl = 'https:' == document.location.protocol,
s = document.getElementsByTagName('script')[0],
qc = document.createElement('script');
qc.async = true;
qc.src = (ssl ? 'https://secure' : 'http://edge') + '.quantserve.com/quant.js';
s.parentNode.insertBefore(qc, s);
_qevents.push({ qacct: "p-c1rF4kxgLUzNc" });
/**/
var sc = document.createElement('script');
sc.async = true;
sc.src = (ssl ? 'https://sb' : 'http://b') + '.scorecardresearch.com/beacon.js';
s.parentNode.insertBefore(sc, s);
_comscore.push({ c1: "2", c2: "17440561" });
})();
</script>
</body>
</html>